FEDERAL JURISDICTION

Lakukan tugas rumah & ujian kamu dengan baik sekarang menggunakan Quizwiz!

An actress brought suit in federal district court in State B against her business manager alleging fraud. The suit was properly brought under diversity jurisdiction. The actions constituting the alleged fraud had occurred in State A, where the business manager resided. The actress resided in State B. Under the law of State A, the actress would be entitled to a jury to hear her claim. Under the law of State B, no jury is allowed in such matters. State B's conflict-of-law rules provide that procedural matters are controlled by the law of the forum state, while substantive matters are to be controlled by the law of the place where the alleged wrong occurred. The actress has demanded a jury to hear the matter and the business manager has objected. How should the federal district court rule with regard to the actress's right to a jury?

A Allow a jury, because the law of the location of the alleged wrong controls. BAllow a jury, because the right to a jury is considered substantive, rather than procedural. CDeny a jury, because the right to a jury is considered procedural, rather than substantive. DDeny a jury, because the matter is equitable, rather than legal. Answer choice C is correct. In diversity actions, a federal district court is generally bound by the conflict-of-law rules of the state in which the district court is located. Questions about the right to a jury trial are considered procedural and are determined by the law of the forum state.

The plaintiff, a State N citizen, properly invokes a State M federal court's diversity jurisdiction in a tort suit against the defendant, an airplane manufacturer based in State M. The plaintiff credibly alleges that he was severely injured when the defendant's airplane crashed as a result of an improperly installed engine part. During discovery, the plaintiff learns that an employee of the defendant who installed engine parts at the time the plane was manufactured was an alcoholic whose drinking may have impaired his work. The defendant fired the employee before the plane crash that injured the plaintiff. What discovery device may the plaintiff use to obtain more information from the former employee?

A An oral deposition. B An interrogatory. C A physical examination. D A request for admission. Answer choice A is correct. A plaintiff may take an oral deposition of a nonparty witness, such as the former employee, as long as the nonparty is given proper notice. By contrast, all of the other devices listed (interrogatories, physical exams, and requests for admission) can be used only against a party. Because the former employee no longer works for the defendant, these devises cannot be used to obtain information from him.

Two business partners, who were citizens of neighboring states, entered into an agreement in which one partner would pay 70% of the initial startup costs of a new solar energy business, while the other would pay 30% and repay 20% to the other partner after two years. The parties signed a promissory note outlining this agreement. After the terms outlined in the agreement had passed, the debtor-partner had not paid anything to the creditor-partner, so the creditor-partner appropriately filed suit in federal district court based on diversity jurisdiction. The forum state's partnership statute contained substantive and procedural provisions that varied somewhat from a federal statute that regulated certain lending in the solar energy industry, though the existence of an actual conflict was dependent upon the facts of the matter. How should the court proceed?

A Apply federal law, because there is a federal law that addresses the disputed issue. BApply federal law, because there is a conflict between how the matter is handled. CEvaluate the facts to determine whether a conflict exists. DApply the law of the forum state regardless of whether there is a federal law that addresses the disputed issue. Answer choice C is correct. When the court is faced with a substantive or procedural question for the purpose of applying the Erie doctine, the district court should determine whether there is a conflict between state and federal law with respect to the issue before the court. If there is no conflict between state and federal law, then the analysis does not need to proceed any further because the court can apply both state and federal law. Here, it is unclear whether there is a conflict, so the court must determine whether there is, in fact, a conflict before proceeding with applying either law

A businessman was forced to file for bankruptcy in federal court after a stock portfolio manager he hired negligently lost the businessman's fortune. In the state where the businessman resided, a specific state statute governed all relationships and obligations between debtors and creditors. The state statute contained provisions that regulated how a debtor would discharge debts in the event of a default. Many of the statute's provisions conflicted with federal bankruptcy laws because they afforded different procedures and prescriptive periods than those required for discharging debts in bankruptcy. How should the federal court handle the conflict between the state and federal provisions?

A Apply the state substantive law only, because this conflict arises out of debtor-creditor relations. BApply the state procedural law only, because there is a conflict between federal and state law. CApply federal substantive and procedural law, because this is a bankruptcy action. DApply federal procedural law only, because all procedural issues are addressed by federal law. Answer choice C is correct. If the action is a federal-question claim, then federal substantive and procedural law will control. Bankruptcy actions fall under the purview of federal law, so federal substantive and procedural law will apply here.

A plaintiff filed a breach of contract action based on diversity jurisdiction in federal district court. In her answer, the defendant alleged that she was not liable to the plaintiff due to a novation. The plaintiff did not reply to this allegation and the court did not order the plaintiff to do so. How should the court treat the defendant's novation allegation?

A As admitted by the plaintiff, because the plaintiff did not respond to this allegation. BAs admitted by the plaintiff, because an allegation, other than one that relates to the amount of damages, is deemed admitted if not denied. CAs denied by the plaintiff, because a party is not required to respond to an allegation contained in an opposing party's pleading. DAs denied by the plaintiff, because the plaintiff was not required to respond to the defendant's pleading. Answer choice D is correct. Under Rule 7(a)(7), a reply to the defendant's answer is made only when the plaintiff is ordered to do so by the court. Thus, the plaintiff is deemed to deny any allegations in the defendant's answer, including any affirmative defenses, if not ordered to reply.

A plaintiff brought an action for negligence in federal district court in State A under diversity jurisdiction to recover for injuries allegedly sustained in an accident that occurred while he was a passenger on board a cruise ship in a port in State B. The defendant in the action was a State C corporation that owned the cruise ship. At the time of the alleged negligence, the ship was docked in the State B port after finishing a voyage from a port in State A. For several years prior to the voyage on which the plaintiff was injured, the cruise ship had brought passengers to visit three ports in State A and had most recently spent several weeks in repair at a shipyard in State A. The State A long-arm statute authorizes personal jurisdiction over a nonresident defendant if the defendant "does business" in State A. The phrase "does business" has been interpreted by the highest court of State A to reach as far as is constitutionally permissible. If the defendant timely moves to dismiss the action for lack of personal jurisdiction, how is the district court likely to rule?

A Deny the motion, because the defendant has impliedly consented to the court's jurisdiction. B Deny the motion, because the defendant could reasonably anticipate being taken to court in State A. C Grant the motion, because the accident occurred in State B. D Grant the motion, because the defendant's contacts with State A have not been systematic and continuous. The defendant could reasonably anticipate being taken to court in State A with respect to an accident occurring on a sea voyage that originated in State A.

An employer properly filed a complaint in federal district court seeking an injunction to enforce a covenant not to compete against a former employee. The former employee properly filed an answer that contained a counterclaim for damages stemming from alleged unpaid wages. The employer timely filed a demand for a jury trial with regard to the counterclaim. The former employee timely filed a motion challenging this demand. How should the court rule on the former employee's motion?

A Deny the motion, because the employer is seeking an injunction. BDeny the motion, because the former employee is seeking damages. CGrant the motion, because the employer is not seeking damages. DGrant the motion, because only a defendant may demand a jury trial. Answer choice B is correct. Any party may make a jury trial demand with regard to an action at law, which includes any claim for damages. The right to trial by jury is evaluated for each claim. Because the former employee is seeking damages in the counterclaim, the employer is entitled to a jury trial on this issue. Consequently, the court should deny the former employee's motion that challenged the employer's demand for a jury trial on the counterclaim.

A plaintiff sued a defendant in federal district court for the western district of a state. The plaintiff, who lived in the same city as the court was located, asserted a cause of action that arose under federal law. The defendant, a lifelong resident of a town located in the eastern federal judicial district of the state, was served with the complaint and summons at his home. The federal district court for the western district, which issued the summons, is located more than 100 miles from the defendant's home. The defendant timely filed a motion to dismiss the complaint for lack of personal jurisdiction. How should the court rule on this motion?

A Deny the motion, because the plaintiff and the defendant live in different judicial districts. BDeny the motion, because the defendant was a lifelong resident of the forum state. CGrant the motion, because the forum court was located more than 100 miles from the defendant's home. DGrant the motion, because the plaintiff and defendant are residents of the same state. Answer choice B is correct. The court has personal jurisdiction over the defendant because the defendant, as a lifelong resident of the forum state, is domiciled in the forum state. Domicile of the defendant in the forum state is a sufficient basis on which a federal court in that state can exercise personal jurisdiction.

A plaintiff properly filed a complaint in federal district court under diversity jurisdiction against an electrician. According to the complaint, the electrician allegedly negligently performed electrical work in a building owned by the plaintiff, which resulted in a fire that caused $150,000 in damages to the building. The electrician served an answer to the complaint. Twelve days later, the electrician filed with the court and served, without leave of the court, a summons and third-party complaint on his partner. The electrician and his partner are citizens of different states. According to this complaint, if the electrician is liable to the plaintiff, the partner is liable to the electrician for half of the damages. The partner has moved to dismiss this claim against him. How should the court rule?

A Dismiss the claim, because impleader is only appropriate if the third party is liable for the full amount of the claim against the defending party. BDismiss the claim, because the court lacks subject matter jurisdiction over this third-party claim. CAllow the claim, because the electrician has timely asserted the claim against his partner, and the court has supplemental jurisdiction. DAllow the claim, because the court has diversity jurisdiction over the claim. Answer choice C is correct. A defendant may implead a third party at any time after the complaint has been filed by serving a summons and third-party complaint on the third party. The defendant (i.e., the third-party plaintiff) may do so without the court's permission if he files the third-party complaint within 14 days after serving his answer to the plaintiff's complaint. Here, since the electrician filed his third-party complaint 12 days after service of his answer, the electrician did not need the court's permission to file the third-party complaint

An insurer issued a homeowner's policy that covered collectible historical documents. The documents, valued at and insured for $70,000, were lost in a fire that destroyed the home. Prior to paying the homeowner, the insurer received a demand from a third party that the insurer pay the $70,000 to the third party. The third party contended that the documents had been stolen from the third party. The insurer and third party were citizens of the same state, while the homeowner was a citizen of a different state. The insurer would like to file a federal statutory interpleader action in the federal district court for the state of which the homeowner is a citizen. Although this state has a long-arm statute, neither that statute nor the general service of process rules would permit service of process on the third party. The insurer does not wish to deposit the $70,000 with the court or post a bond pending resolution of this matter. Of the following, which is a valid reason for advising against the filing of a federal statutory interpleader action in the federal district court?

A Diversity of citizenship does not exist between the insurer and the third party. BThe amount in controversy does not exceed $75,000. CNeither the long-arm statute nor the general service of process rules of the state in which the action would be filed permit service of process on the third party. DThe insurer does not wish to deposit the $70,000 with the court or post a bond pending resolution of the matter. Answer choice D is correct. In order to file a federal statutory interpleader action, the stakeholder must be willing to either deposit the property at issue with the court or to post a bond in an appropriate amount. Here, because the insurer does not wish to deposit the $70,000 with the court or post a bond, this deposit requirement provides a valid reason for advising against filing a federal statutory interpleader action.

The plaintiff, a State D citizen, is a shareholder of a corporation incorporated and having its principal place of business in State D. She brought an action in the U.S. District Court in State D requesting an injunction against the corporation, which allegedly was violating a State D law that required corporations to invest only in lawful commercial paper and that gave shareholders a right to sue over alleged violations of this law. The plaintiff claims that the defendant corporation had been purchasing commercial paper that had been issued by the federal government in violation of the U.S. Constitution. The corporation denies this claim and moves to dismiss on the ground that the U.S. District Court lacks jurisdiction. What would the court be most likely to do?

A Exercise general federal-question jurisdiction because federal law created the plaintiff's cause of action. BExercise general federal-question jurisdiction because the plaintiff has a real and substantial issue of federal law and her right to relief depends upon the resolution of this issue. CDismiss for lack of jurisdiction because the plaintiff's claim arose under state law because state law created her cause of action, and there is no diversity of citizenship. DDismiss for lack of jurisdiction because shareholders cannot sue their corporations in federal court, except for violations of federal securities law. Answer choice B is correct. When state law creates a cause of action, a federal court can nonetheless exercise general federal-question jurisdiction if the complaint raises a real and substantial issue of federal law, and the outcome necessarily depends on resolving this federal issue. Here the plaintiff's complaint raises a genuine and substantial federal law issue (the constitutionality of the federal commercial paper), which must be resolved to decide her request for an injunction.

A plaintiff properly brought a civil action in federal district court, sitting in diversity jurisdiction, seeking damages for negligence against a corporate defendant. The trial concluded on March 13 with a verdict for the defendant. The court entered judgment for the defendant on March 14. The plaintiff filed a motion for new trial on March 15. The motion was denied by a signed written order entered on March 24. Which of the following must the plaintiff do to commence an appeal?

A File a notice of appeal within 30 days after March 14th. BFile a notice of appeal within 60 days after March 14th. CFile a notice of appeal within 30 days after March 24th. DFile a notice of appeal within 60 days after March 24th. Answer choice C is correct. When a motion for a new trial is filed, the time for giving notice of appeal will run from the entry of the order denying the new trial. The entry of the order denying the last post-judgment motion starts the 30-day time period for filing an appeal.

A decedent, at death, was the owner of record of a $52,000 account at a state savings bank. As permitted by state law, the decedent's nephew was named in the account records as the person to be paid upon the decedent's death. After the decedent's death, the bank received by mail a documented claim of a creditor of the decedent, claiming that the decedent, shortly before her death, had assigned the account to the creditor. The following day, the nephew appeared at the bank and demanded the $52,000 in the account. The bank has doubts as to whether the decedent's signature on the assignment is valid. The creditor and bank are citizens of the same state as the decedent was at the time of her death. The nephew lives in an adjoining state. The bank filed a statutory interpleader action with the local federal district court. The nephew has filed a motion to dismiss based on lack of subject matter jurisdiction. The bank opposes this motion. How should the court rule on the motion?

A For the nephew, because the amount in controversy fails to meet the jurisdictional minimum. BFor the nephew, because there is not diversity of citizenship between the bank and creditor. CFor the bank, because there is no amount-in-controversy requirement for a statutory interpleader action. DFor the bank, because the probate exception to diversity jurisdiction does not apply. Answer choice D is correct. While federal courts generally do not exercise diversity jurisdiction over probate matters, this restriction is narrowly construed and primarily applies to the probate of a will or the administration of a decedent's estate. Here, the applicable state law recognizes the validity of a paid-on-death account, which permits the owner of the bank account to designate a person to take the account upon the owner's death. Since this property does not pass by the decedent's will and is not included in the decedent's estate, the action does not fall within the probate exception to diversity jurisdiction. Since, as discussed with regard to answer choices A and B, the court otherwise has diversity jurisdiction, the court should deny the nephew's motion to dismiss.

An employee sued her employer in the federal district court in their home state. The plaintiff-employee claimed that the defendant-employer fired her because of her gender, thereby violating a federal statute granting victims of such discrimination a right to sue in federal court. The plaintiff's complaint included an unrelated claim that, many months before her firing, the defendant violated state law by breaching a contract to sell her concert tickets. The defendant's answer contained two key provisions. First, he contended that he did not fire the plaintiff because of her gender, but rather because she routinely took two-hour lunch breaks during which she consumed alcohol, which resulted in the significant impairment of her work performance. Second, the defendant asserted a counterclaim alleging that the plaintiff's lengthy lunch breaks and drinking breached her employment contract under state law. The defendant has moved to dismiss the plaintiff's state-law contract claim involving the concert tickets on jurisdictional grounds. In response, the plaintiff has moved to dismiss the defendant's state-law counterclaim on jurisdictional grounds. How should the court rule on these motions?

A Grant both motions. B Deny both motions. C Grant the defendant's motion, but deny the plaintiff's motion. D Grant the plaintiff's motion, but deny the defendant's motion. C Answer choice C is correct. The federal court should grant the defendant's motion because it has no jurisdiction over the state-law contract claim concerning the tickets. This state claim cannot be entertained by the court through the exercise of supplemental jurisdiction because the claim did not arise out of the same factual predicate as the federal discrimination claim (the firing), but rather involves an unrelated transaction. In addition, the court lacks diversity jurisdiction because both the plaintiff and defendant are from the forum state. Although a plaintiff may join independent claims against a defendant under the permissive joinder rule (Rule 18), the court must have subject matter jurisdiction over these claims, and such jurisdiction is absent. On the other hand, the court should deny the plaintiff's motion to dismiss, even though the court lacks diversity jurisdiction over the defendant's state law claim. The defendant is required to bring the counterclaim that Plaintiff's lunch breaks and drinking violated state law because that counterclaim arose out of the same factual occurrence that is the subject of the plaintiff's federal claim (the firing). This commonality warrants the exercise of supplemental jurisdiction. Accordingly, answer choices A, B, and D are incorrect.

An auction company held an auction of a rare historical document consigned to the company for sale by a private individual. At the auction, the document sold to a partnership for $50,000. Before the document was delivered to the partnership, a library contacted the auction company and asserted ownership of the document. The auction company filed a federal statutory interpleader action in federal district court, naming the partnership, the individual partners, and the library as defendants. State law permits a partnership to be sued as a separate entity from its individual partners. The forum court is located in the district where the auction company has its principal place of business and in the state in which the auction company is incorporated. The partnership consists of two partners. Partner A, the partner who bid on behalf of the partnership at the auction, is domiciled in another district of the same state in which the forum court is located. Partner B is domiciled in a neighboring state and has no personal contacts with the state in which the forum court is located. The library is domiciled in a third state. Partner B has filed a motion to dismiss the action based on lack of subject matter jurisdiction, personal jurisdiction, and improper venue. How should the court rule on this motion?

A Grant it, because neither the amount-in-controversy nor the diversity of citizenship requirement for subject matter jurisdiction has been satisfied. B Grant it, because Partner B has no contacts with the forum state, so the court does not have personal jurisdiction over Partner B. C Grant it, because no claimant resides in the same district as the forum court, so venue is improper. D Deny it, because the court has both subject matter and personal jurisdiction and venue is proper. Answer choice D is correct. As discussed in detail below with regard to the incorrect answer choices, the court has both subject matter jurisdiction and personal jurisdiction and venue is proper. Answer choice A is incorrect. It is true that this action does not satisfy the requirements for a general diversity-based action. The amount-in-controversy ($50,000) does not exceed the $75,000 threshold required for a general diversity-based action. In addition, diversity of citizenship does not exist for such an action because the plaintiff, the auction company, is a citizen of the same state as one of the partners. Although the partnership was named as a defendant, for purposes of the determining subject matter jurisdiction the partnership is considered to be a citizen of each state in which a partner is a citizen. However, because this is a federal statutory interpleader action, the value of the property at issue must only exceed $500, which is the case with this document, and any two claimants must be citizens of different states. Because neither partner is domiciled in the state in which the library is domiciled, the minimum diversity required under a federal interpleader action has been satisfied. Consequently, the forum court has subject matter jurisdiction over this action. Answer choice B is incorrect. While Partner B is not domiciled in the forum state and has no personal contacts with that state, nationwide personal jurisdiction over a party exists with regard to an action brought under the federal interpleader statute. Answer choice C is incorrect. While none of the claimants reside in the district in which the forum court is located, the partnership, an entity with the capacity to sue and be sued in its common name under applicable law, is deemed to reside in any judicial district in which such defendant is subject to the court's personal jurisdiction. Because, as noted with respect to answer choice B, the federal interpleader statute provides for nationwide personal jurisdiction over a party, venue is proper in the forum court.

A tenant properly filed a complaint under state law against his landlord in federal district court under diversity jurisdiction, alleging that the landlord's negligence in failing to repair the stairs in a common area of the apartment building resulted in the tenant falling and sustaining significant injuries. After the landlord served his answer, the tenant moved for summary judgment. In support of his motion, the tenant submitted affidavit from three other tenants in which each stated that she had told the landlord prior to the accident that the stairs needed to be fixed. In opposition to the motion, the landlord submitted an affidavit from the maintenance worker employed by the landlord, stating that the stairs were in good condition on the day of the tenant's accident. How should the court rule on the motion for summary judgment?

A Grant the motion, because in viewing the evidence in the light most favorable to the tenant, the tenant is entitled to judgment as a matter of law. BGrant the motion, because the tenant presented more evidence in support of the motion. CDeny the motion, because the tenant did not meet the burden of production. DDeny the motion, because a reasonable jury could return a verdict in favor of the landlord. Answer choice D is correct. A motion for summary judgment must be granted if the pleadings, the discovery and disclosure materials on file, and any affidavits show that there is no genuine dispute as to any material fact and that the movant is entitled to judgment as a matter of law. A genuine dispute of material fact exists when a reasonable jury could return a verdict in favor of the nonmoving party.

A former employee filed a complaint in federal district court under diversity jurisdiction seeking actual and punitive damages from her former employer based on a state law claim. Under the state's law, unlike federal law, punitive damages are determined by the court rather than a jury. The complaint contained a demand for a jury trial on all issues. Prior to filing an answer or a pre-answer motion to dismiss, the former employer timely filed a motion to strike the jury trial demand with regard to the determination of punitive damages. How should the court rule on this motion?

A Grant the motion, because state law governs with regard to punitive damages in an action based on diversity jurisdiction. BGrant the motion, because a plaintiff cannot make a jury trial demand until the defendant has served a responsive pleading or the time for doing so has expired. CDeny the motion, because the right to a jury trial is established under federal law. DDeny the motion, because the plaintiff filed a jury trial demand before the defendant served an answer or a pre-answer motion. Answer choice C is correct. Because the Seventh Amendment provides for a right to a jury trial, federal law governs whether there is a right to a jury trial, even when the action is based on state law and the federal court's jurisdiction is based on diversity. Because the facts indicate that, under federal law, punitive damages are determined by the jury, the former employee can demand a jury trial with regard to the issue of punitive damages. Consequently, the court should deny the former employer's motion to strike the jury trial demand.

A plaintiff sued a defendant for personal injuries resulting from negligence in a federal district court sitting in diversity jurisdiction. The plaintiff alleged that she had been crossing at a crosswalk when the defendant ran a red light at an intersection and hit her with his truck, causing injuries. Without any showing of good cause other than the need to verify the plaintiff's injuries, the defendant filed a motion to compel the plaintiff to submit to a physical examination. The plaintiff has opposed the examination as intrusive. How should the court rule on the defendant's motion?

A Grant the motion, because such a physical examination is permitted if requested by the defendant, regardless of whether "good cause" is shown. BGrant the motion, because the plaintiff has placed her physical condition into issue. CDeny the motion, because a physical examination may only be ordered for "good cause." DDeny the motion, because the court can only order a physical examination with the consent of the party to be examined. Answer choice B is correct. Although a physical examination may be ordered only for "good cause," the plaintiff here has placed her physical condition into controversy by claiming personal injury damages, which is sufficient "good cause" for the court to order a physical examination.

A plaintiff properly sued a defendant in federal district court in State A under diversity jurisdiction to enforce a promissory note. The plaintiff was a citizen of State A and the defendant was a citizen of State B. The defendant sought to implead her business associate as a third-party defendant in the case, asserting that her business associate would be required to indemnify her if she was held liable to the plaintiff. The business associate was a citizen of State C, and has never had any contacts with State A. State A's long-arm statute does not provide for out-of-state service on the business associate. The defendant personally served his business associate with a summons and complaint at the associate's home in State C, which is located 80 miles from the forum court. The business associate has moved to dismiss the third-party complaint for lack of personal jurisdiction. How should the court rule on the business associate's motion?

A Grant the motion, because the court does not have personal jurisdiction over the business associate. BGrant the motion, because State A law does not allow for out-of-state service on the business associate. CDeny the motion, because the court has personal jurisdiction over the business associate as a third-party defendant. DDeny the motion, because the court's personal jurisdiction over the defendant automatically extends to the business associate. Answer choice C is correct. Under Rule 4(k)(1)(B), a federal court has personal jurisdiction over a third-party defendant, joined under Rule 14, who is served within a U.S. judicial district and not more than 100 miles from the court from where the summons is issued (the federal district court), even if state law would otherwise not permit such service. Here, the business associate is impleaded as a third-party defendant pursuant to Rule 14, and is being served at his home, which is located within 100 miles of the federal district court in State A. Accordingly, under the so-called "bulge provision," the court has personal jurisdiction over the business associate.

A customer who was injured by a fall at a retailer's store sued the retailer for negligence. The action was properly brought in federal district court based on diversity jurisdiction. After the trial began, the customer, due to a death in the family, requested an extended continuance. The court granted the customer's request. Due to the continuance, the court dismissed, for good cause, a juror who had prepaid a vacation trip that was scheduled to begin during the period of the continuance and run through the anticipated completion of the trial. The dismissal left the jury with six members. After the case was given to the jury for deliberation, a juror took ill. The court dismissed this juror. Given the option of a new trial, both the customer and the retailer agreed to a five-person jury. After the jury returned a unanimous verdict for the customer, the retailer moved to vacate the verdict based on the size of the jury. How will the court likely rule on the retailer's challenge?

A Grant the motion, because the court may not dismiss a juror once deliberations have begun. BGrant the motion, because a verdict returned by a jury of less than six members violates the Federal Rules of Civil Procedure. CDeny the motion, because the parties stipulated to a five-person jury. DDeny the motion, because the court's dismissal of two jurors was for good cause. Answer choice C is correct. A case may be tried before and a verdict returned by less than a six-member jury, if the parties so stipulate. Here, both the customer and the retailer agreed to a five-person jury.

A plaintiff corporation, incorporated and headquartered in State X, sued a defendant corporation, headquartered in a foreign country, in federal district court in State X for a common-law breach-of-contract claim, asserting diversity jurisdiction. The parties negotiated and executed the contract in the foreign country, and performance was also to occur there. The defendant corporation has insufficient contacts with State X and with any other U.S. state to justify the exercise of jurisdiction by a state court over the defendant. If the defendant moves to dismiss the action based on lack of personal jurisdiction, how should the court rule?

A Grant the motion, because the defendant corporation does not have sufficient contacts with the forum state. BGrant the motion, because diversity between the plaintiff and the defendant does not exist. CDeny the motion, because the plaintiff cannot bring the contract action against the defendant in any other state court. DDeny the motion, because the federal district court has personal jurisdiction over a foreign defendant if the plaintiff is a U.S. citizen.

A plaintiff sued a defendant in federal district court. The plaintiff was domiciled in the forum state; the defendant was domiciled in another state. The plaintiff asserted two unrelated causes of action against the defendant. One was based on a breach of contract, for which the plaintiff sought $60,000 in damages. The other was a personal injury claim for which $25,000 in damages was sought. The defendant, who was served with process while in the forum state, filed a timely motion to dismiss based on the lack of subject-matter jurisdiction. How should the court rule?

A Grant the motion, because the plaintiff's claims are unrelated and neither exceeds $75,000. B Grant the motion, because the plaintiff is domiciled in the forum state. C Deny the motion, because the defendant was served with process in the forum state. D Deny the motion, because diversity jurisdiction exists. Answer choice D is correct. In order for subject-matter jurisdiction based on diversity to exist, the amount in controversy must exceed $75,000 and the plaintiff and defendant must be citizens of different states. When there is only one plaintiff and one defendant, all of the plaintiff's claims against the defendant, including unrelated claims, are aggregated for purposes of determining whether the amount-in-controversy requirement has been satisfied.

A jury found for the plaintiff in a defamation action in federal district court against a newspaper publisher. Following the verdict, the newspaper publisher moved for a new trial on the grounds that the verdict was against the weight of the evidence. The court granted the motion. The plaintiff wants to immediately appeal the court's order granting the new trial. Which of the following is most accurate concerning immediate appeal of the court's order?

A Immediate appeal is not available because the basis of the motion was that the verdict was against the weight of the evidence. BImmediate appeal is precluded by the final judgment rule. CImmediate appeal is available if the appeal is filed with the circuit clerk within 14 days after the order is entered. DThe plaintiff may appeal the order as of right. Answer choice B is correct. An order for a new trial is not appealable because it is not a final judgment. A party who wants to raise on appeal the grant of a new trial must wait until the new trial has occurred and resulted in a final judgment. The party may then appeal from that judgment and raise as an issue the order for the new trial.

An Internet startup company began marketing a line of products under a trade name that was identical to a manufacturer's protected trademark. The manufacturer filed an action in federal district court for injunctive relief against the startup company, seeking to put a stop to the company's use of the protected trade name. The district court granted a preliminary injunction in favor of the manufacturer and scheduled a hearing to consider a permanent injunction. The startup company wants to immediately appeal the preliminary injunction. Which of the following is most accurate concerning the startup company's potential appeal?

A Immediate appeal is precluded by the final judgment rule. B Immediate appeal is precluded because the relief sought in the action is equitable. C Immediate appeal is allowed by right. D Immediate appeal is allowed only with the approval of the district court. Answer choice C is correct. While most interlocutory orders are not immediately appealable, certain equitable orders are reviewable immediately as a matter of right, including an order granting an injunction.

The distracted driver of a powerboat struck and seriously injured two water skiers who, at time of the accident, were performing a stunt in which one was on the shoulders of the other. The unrelated skiers, who each suffered damages in excess of $100,000, filed a joint complaint based on negligence in a federal court located in the district in which the accident occurred. One of the skiers was a citizen of the forum state. The driver-defendant and the other skier were citizens of a neighboring state. The forum state has a long-arm statute that permits the exercise of jurisdiction to the extent permissible under the Due Process Clause of the Fourteenth Amendment. Upon which of the following grounds can the driver most likely successfully move for dismissal of the claim asserted by the skier who is not a citizen of the forum state?

A Improper joinder B Lack of subject matter jurisdiction C Lack of personal jurisdiction D Lack of venue Answer choice B is correct. The federal district court has neither diversity nor supplemental jurisdiction over the claim of the non-forum skier. This claim satisfies the amount-in-controversy requirement since it exceeds $75,000, but it does not satisfy the diversity-of-citizenship requirement because the non-forum skier and the defendant are both citizens of the same state.

A business owner brought an action for breach of contract under state law against an architect in federal district court. The federal district court was not located in the state in which the architect resided or in the state in which the contract was executed or performed. The business owner's complaint alleged $75,000 in damages and the business owner and architect were citizens of different states. The business owner himself served the architect with the summons and complaint. The architect had no contacts with the forum state that would give rise to personal jurisdiction. The architect served an answer in which he specifically denied the business owner's allegations. Two weeks after serving the answer, the architect sought to have the complaint dismissed. On which of the following grounds will the architect be successful in having the claim dismissed?

A Improper venue BInsufficient service of process CLack of subject-matter jurisdiction DLack of personal jurisdiction Answer choice C is correct. A federal court may exercise diversity jurisdiction over a state law claim if (i) no plaintiff is a citizen of the same state as any defendant and (ii) the amount in controversy in the action exceeds $75,000. The defense of lack of subject-matter jurisdiction may be raised at any time, even on appeal. In this case, there was no basis for federal diversity jurisdiction because the amount in controversy did not exceed $75,000, a common trick used by the bar examiners. Therefore, the architect can seek dismissal based on lack of subject-matter jurisdiction.

A plaintiff brought a state law action for medical malpractice against a defendant in federal district court under diversity jurisdiction. The plaintiff's complaint alleged only that the defendant's negligent conduct caused the plaintiff's damages. The complaint did not include any specific factual allegations regarding the defendant's conduct. The defendant has not yet filed an answer to the complaint. In response to the complaint, which of the following motions would be most appropriate for the defendant to file?

A Motion to strike BMotion for summary judgment CMotion for judgment on the pleadings DMotion to dismiss for failure to state a claim upon which relief can be granted Answer choice D is correct. Under Rule 12(b)(6), a claim for relief can be dismissed if it either fails to assert a legal theory of recovery that is cognizable at law or fails to allege facts sufficient to support a cognizable claim. In reviewing a motion to dismiss under Rule 12(b)(6), the court must identify and reject legal conclusions unsupported by factual allegations. This includes mere conclusory statements and assertions devoid of facts.

A jeweler who is a citizen and resident of a foreign country brought some jewels into the United States to sell at a convention. A buyer purchased the jewels from the jeweler, whom he had not previously met, but whom he knew by reputation to be an honest businessperson. Subsequently, the buyer discovered that the jeweler had misrepresented the quality of the jewels. The buyer has filed an action in the federal court for the district in which he resides against the jeweler based on a state law claim of misrepresentation. The convention had been held in another state. The jeweler has timely filed a motion to dismiss this action based on lack of subject matter and personal jurisdiction, as well as improper venue. The court properly determined that it has subject matter and personal jurisdiction. Should the court grant the motion as it relates to improper venue?

A No, because a nonresident of the United States may be sued in any judicial district. BNo, because the action is based on diversity jurisdiction, not federal-question jurisdiction. CYes, because the action is based on a state law claim. DYes, because the cause of action arose in another state. Answer choice A is correct. A defendant who is not a resident of the United States may be sued in any judicial district. Because the jeweler is a resident of a foreign country, the jeweler may be sued in the federal district court for the district in which the buyer resides.

In a negligence action properly before a federal district court sitting in diversity, the court submitted the case to the jury. The jury's decision, which took the form of a written special verdict, was read aloud by the court clerk in open court. The verdict stated that both parties were negligent and that both parties' negligence proximately caused the plaintiff's injuries. The verdict also stated that the plaintiff had suffered damages of $1 million and was 10% at fault for his injuries. The court then asked the jury collectively if this was their verdict and they responded in unison, "Yes." The defendant requested that the jury be polled. When questioned individually, a juror tearfully stated that the verdict was not her verdict because she did not believe that the defendant had been negligent. Upon further questioning, she maintained this position. The other seven jurors affirmed the verdict. The defendant moved for a new trial. Is the court likely to grant the defendant's motion?

A No, because at least six jurors agreed with the verdict. B No, because a juror cannot recant a special verdict once it has been read aloud by the court clerk in open court. C Yes, because the court must order a new trial when polling the jury reveals that the verdict is not unanimous. D Yes, because one juror did not affirm the special verdict as hers. Answer choice D is correct. Under the federal rules, the verdict of a civil jury must be unanimous unless the parties otherwise agree. Here, one juror, when questioned individually, denied that the special verdict was her verdict. Consequently, the court may order a new trial.

An insurer, having paid the claim of the owner of a building that was destroyed by fire, properly filed an action in federal district court sitting in diversity against a tenant in the building. The insurer, asserting that it was subrogated to the rights of the owner of the building, sued the tenant to recoup the amount the insurer had paid to the building owner. The insurer asserted that the tenant's negligence caused the fire. A second tenant timely filed a motion to intervene in this action, asserting a claim against the first tenant for its own losses that it suffered as a consequence of the fire. The second tenant's losses exceeded $100,000. Both tenants are citizens of the same state. Does the court have subject matter jurisdiction over the second tenant's claim against the first tenant?

A No, because both tenants are citizens of the same state. BNo, because the second tenant's intervention is not as of right. CYes, because the second tenant's claim satisfies the amount-in-controversy requirement. DYes, because the court has supplemental jurisdiction over the second tenant's claim. Answer choice A is correct. In order for a federal court to have subject matter jurisdiction over a claim based on diversity jurisdiction, both the amount-in-controversy and the diversity-of-citizenship requirements must be met. Here, although the second tenant's claim satisfies the amount-in-controversy requirement, it does not satisfy the diversity-of-citizenship requirement because the second tenant and the first tenant are both citizens of the same state.

A bankruptcy trustee filed a complaint in federal district court pursuant to a federal statute to recover property transferred by the debtor after the filing of the bankruptcy petition to two transferees for less than reasonably equivalent value. According to the complaint, one defendant had received real property valued at $250,000, and the other defendant, in an unrelated transaction, had received personal property valued at $25,000. Each defendant received the property as a gift from the debtor. The recipient of the personal property has filed a motion to dismiss due to misjoinder of parties. Should the court grant this motion?

A No, because each transferee's liability is based on the same cause of action. BNo, because the value of the real property transferred to the non-moving defendant exceeded the amount-in-controversy requirement. CYes, because the two transfers were unrelated. DYes, because the value of the personal property transferred to the movant was less than the amount-in-controversy requirement. Answer choice C is correct. In order for joinder of defendants to be proper, any right to relief must arise out of the same transaction, occurrence, or series of transactions or occurrences. Here, the transfer of the personal property was unrelated to the transfer of the real property. Consequently, joinder of both defendants in a single action was improper. In addition, when parties are joined, the relief must be sought from them jointly, severally, or in the alternative. While relief can be sought from each defendant with respect to the property that that defendant received from the debtor, neither defendant has responsibility with respect to the property received by the other defendant from the debtor.

A plaintiff properly filed a complaint in federal district court seeking damages from two defendants. The first defendant properly filed a cross-claim against the second defendant. The second defendant has a claim against the first defendant that arises out of the same transaction as the original complaint. Is the second defendant required to bring this claim in the current action?

A No, because it is a permissive cross-claim. BNo, because it is a permissive counterclaim. CYes, because it is a compulsory cross-claim. DYes, because it is a compulsory counterclaim. Answer choice D is correct. If a party files a claim, a counterclaim, or a cross-claim against another party, the second party must file any claim that that party has against the first party that arises out of the same transaction or occurrence as the first party's claim. The first defendant is an opposing party with respect to this mandatory claim; accordingly, the second defendant's claim is a compulsory counterclaim. (Note: A cross-claim may properly be pursued in the current action only if it arises out of the same transaction or occurrence as the original complaint. Consequently, a claim made in response to a cross-claim that arises out of the same transaction or occurrence as the original complaint also arises out of the same transaction or occurrence as the cross-claim.)

An engineer and a corporation entered into a contract. After the engineer failed to perform as required under the contract, the corporation filed a complaint for breach of contract in federal district court based on diversity jurisdiction. The corporation was incorporated and had its principal place of business in the forum state. The contract had been executed and performance was required to have taken place in a neighboring state of which the engineer was a lifelong resident. Instead of answering the complaint, the engineer served a motion to dismiss based on a lack of personal jurisdiction. Prior to the court ruling on the engineer's motion to dismiss, can the corporation dismiss the action without the engineer's agreement or the approval of the court?

A No, because it is likely that the court lacked personal jurisdiction over the engineer. BNo, because the engineer failed to perform as required under the contract. CYes, because a plaintiff is free to dismiss an action until it is set for trial. DYes, because the engineer had not served an answer or summary judgment motion. Answer choice D is correct. A plaintiff may dismiss an action without court approval or the agreement of the other parties prior to the service of an answer or summary judgment motion. A motion to dismiss for lack of personal jurisdiction is not a summary judgment motion.

A defendant in a civil action in federal district court filed a complaint against a third-party defendant for contribution. The third-party defendant moved for dismissal based on a failure to state a claim upon which relief could be granted. The court granted the motion to dismiss, without prejudice. The defendant wants to appeal immediately, contending that the trial judge erred in dismissing the complaint against the third-party defendant. Is the court's order immediately appealable?

A No, because of the collateral-order doctrine. BNo, because of the final judgment rule. CYes, because the order granted a motion to dismiss. DYes, because the order dismissed a third-party defendant. Answer choice B is correct. Only a final judgment is subject to immediate appeal. Usually, a judge's 12(b)(6) dismissal is considered a "final judgment" subject to immediate appeal. However, when there are multiple claims and parties, any order or other decision that adjudicates fewer than all of the claims or the rights and liabilities of fewer than all of the parties will not end the action as to any of the claims or parties, and it may be revised at any time before the entry of a judgment adjudicating all of the claims and all of the parties' rights and liabilities. The district court here did not direct entry of a final judgment as to the third-party defendant. It merely dismissed the claim, without prejudice, without directing entry of a final judgment.

A citizen of State Y, who owned land in State Y as a tenant-in-common, brought a partition action with respect to this land in a State Y state court. At the time of the initiation of the partition action, the amount in controversy with regard to this action was $71,000. The defendants, the other tenants-in-common, filed a counterclaim against the plaintiff for an accounting of the profits received by the plaintiff from the property. The counterclaim asserted in good faith that these profits exceed $10,000. The defendants, none of whom was a citizen of State Y, although one was a citizen of Puerto Rico, timely filed a petition for removal of the case to federal court. Does the federal court have subject-matter jurisdiction over this action?

A No, because the amount-in-controversy requirement is not satisfied. BNo, because one of the defendants is not a citizen of a state. CYes, because the aggregated claims meet the amount-in-controversy requirement. DYes, because no defendant is a citizen of State Y. Answer choice A is correct. In determining whether the amount-in-controversy requirement has been satisfied for purposes of determining the existence of diversity jurisdiction, the amount of a defendant's counterclaim may not be aggregated with the plaintiff's claim. In addition, removal jurisdiction cannot be based on a counterclaim that does not independently meet the amount-in-controversy requirement.

A plaintiff properly filed a civil action for breach of contract in a federal district court sitting in diversity. During voir dire, the defendant's attorney questioned a prospective juror about his familial relationship to the plaintiff. The potential juror stated that he was the plaintiff's third cousin, but also stated that he had not had any dealings with the plaintiff in over 20 years. The defendant challenged this prospective juror for cause, due to his potential bias. After questioning the prospective juror about his ability to give a fair trial to all parties and receiving an affirmative reply, the court denied the challenge. The defendant, having exercised three peremptory challenges, sought to excuse the potential juror by exercising a fourth peremptory challenge. Should the court permit the defendant to exercise a fourth peremptory challenge?

A No, because the court had denied the defendant's challenge for cause with respect to this prospective juror. BNo, because the defendant has already exercised his peremptory challenges. CYes, because a party has an unlimited number of peremptory challenges, provided such challenges are not made for racial- or gender-based reasons. DYes, because a court cannot dictate how a party exercises peremptory challenges that are not made for racial- or gender-based reasons. Answer choice B is correct. A party in a federal civil action is entitled to three peremptory challenges. Because the defendant has already exercised three peremptory challenges, the court should deny the defendant's request to exercise a fourth

A company properly filed a diversity action in federal district court against an accountant for malpractice in preparing the corporation's financial statements. The accountant impleaded his malpractice insurer, asserting a claim against the insurer in the event that the accountant was liable to the company. The company then timely filed a third-party claim against the insurer for $90,000 in damages to the company vehicles caused by a falling tree. The insurer and the accountant are citizens of the same state, while the company is a citizen of a different state. The insurer has moved to dismiss this claim due to improper joinder. Should the court grant the insurer's motion?

A No, because the court has subject matter jurisdiction over the company's claim against the insurer. BNo, because a plaintiff may file a third-party claim against a third-party defendant. CYes, because the court lacks jurisdiction over the claim asserted by the accountant against the insurer. DYes, because the company's claim against the insurer does not arise out of the same transaction or occurrence as the original complaint. Answer choice D is correct. A plaintiff may file a third-party claim against a third-party defendant, but the claim must relate to the same transaction or occurrence as the original complaint. Because the company's claim against the insurer does not relate to the company's malpractice claim against the accountant, the company is not permitted to assert this claim as part of the current lawsuit.

A plaintiff properly brought an action based on negligence in federal district court sitting in diversity jurisdiction. At the close of the presentation of the evidence by both parties, both the plaintiff and the defendant filed with the court and served on the opposing party proposed jury instructions. The plaintiff's instructions were silent on the issue of assumption of the risk. Before the parties' closing arguments, the court informed the parties of the jury instructions that it planned to give. With regard to the issue of assumption of the risk, the court rejected the defendant's proposed instruction and substituted its own instruction. The plaintiff objected to the court's proposed assumption-of-the-risk instruction on the record and out of the presence of the jury, but the court overruled the objection. The defendant made no objection. The court then delivered its jury instruction before both parties' closing arguments. Has the court complied with the Federal Rules of Civil Procedure?

A No, because the court must use the defendant's instructions on the issue of assumption of the risk, as the plaintiff did not submit jury instructions on that issue. BNo, because the court instructed the jury before the parties' closing arguments. CYes, because the court is permitted, but not required, to give each party the opportunity to object to the court's jury instructions. DYes, because the court timely informed both parties of the jury instructions it planned to g Answer choice D is correct. Before the parties' closing arguments, the court must inform the parties of the jury instructions it proposes to give and provide the parties with the opportunity to object to such instructions on the record and out of the presence of the jury. When a party has submitted proposed jury instructions, the court must also inform the party of the court's actions with regard to such instructions.

A woman and her daughter were involved in a car accident with another car. The mother sustained serious injuries, but the daughter's injuries were relatively minor. The mother and the daughter filed a joint complaint against the driver of the other car in federal court in the state where the accident occurred. The complaint alleged that the defendant's negligence caused the mother and daughter to sustain physical injuries. The mother asserted damages of $250,000 against the defendant, and the daughter asserted damages of $10,000. The defendant is a citizen of the forum state, and the mother and daughter are citizens of a neighboring state. The defendant filed a motion to dismiss the claims asserted by the daughter, arguing that the court lacked jurisdiction over her claims. May the court exercise jurisdiction over the daughter's claims?

A No, because the daughter's claims do not meet the requirements for diversity jurisdiction. BNo, because the daughter is not a necessary party. CYes, because the court may exercise supplemental jurisdiction over the daughter's claims. DYes, because the daughter's participation is necessary for a just adjudication. Answer choice C is correct. In meeting the amount-in-controversy requirement to satisfy diversity jurisdiction, multiple plaintiffs may aggregate their claims to reach an amount in excess of $75,000, but complete diversity must remain. In this case, the claims asserted by the mother met the requirements for diversity jurisdiction, and there was complete diversity between the plaintiffs and the defendant. Accordingly, the court may exercise supplemental jurisdiction over the daughter's claims.

A plaintiff was involved in a serious car accident with a defendant. The plaintiff sustained damages to her vehicle and significant personal injuries. Two months after the accident, the plaintiff filed a complaint against the defendant in federal district court with subject matter jurisdiction, alleging that the defendant's negligent conduct in failing to stop at a stop sign caused the plaintiff's damages. Twenty days after the defendant served his answer specifically denying the plaintiff's allegations as to the defendant's negligence, the plaintiff moved for summary judgment on the issue of liability. In support of her motion, the plaintiff submitted an affidavit from an eyewitness, stating that he saw the defendant drive through a stop sign immediately before striking the plaintiff's car. This affidavit would be inadmissible at trial as hearsay. The defendant responded to the plaintiff's motion by repeating the denials in his answer. Can the court grant the plaintiff's motion?

A No, because the defendant has specifically denied negligent conduct with regard to the accident. BNo, because the plaintiff relied on an affidavit that would not be admissible at trial. CYes, because there is not a genuine dispute as to the facts. DYes, because the plaintiff's motion was filed within 21 days of service of the answer. Answer choice C is correct. If a motion for summary judgment is properly made and supported, an opposing party may not rely merely on the allegations or denials in her own pleading. The opposing party must set out specific facts showing a genuine dispute for trial. If the opposing party does not so respond, then summary judgment, if appropriate, will be entered against that party. In this case, the plaintiff properly supported her motion for summary judgment with affidavits from a witness and an additional document. Thus, the defendant could not rely merely on the denials in his answer. He should have provided other evidence, such as affidavits, to support his opposition to the motion.

A plaintiff sued a defendant in federal district court following a car accident between the parties that occurred in the state where the federal district court is located. The plaintiff asserted a negligence claim under state law, and in good faith alleged damages of $100,000. At the time of the car accident, both parties were domiciled in the forum state. Following the accident, but before filing the complaint, the plaintiff accepted a full-time job in another state and moved to that state with the intent of living there permanently. After a bench trial, the judge ruled for the plaintiff and awarded $70,000 in damages. The defendant appealed the decision, asserting for the first time that the trial court lacked subject-matter jurisdiction. Is the defendant likely to succeed in his appeal?

A No, because the defendant waived the issue by failing to raise it in the lower court. BNo, because the requirements for diversity jurisdiction were satisfied. CYes, because the plaintiff recovered less than $75,000 in damages. DYes, because the parties were not citizens of different states when the claim arose. Answer choice B is correct. U.S. district courts have diversity jurisdiction when both (i) no plaintiff is a citizen of the same state as any defendant and (ii) the amount in controversy in the action exceeds $75,000. To be a citizen of a state, a person must be a citizen of the United States and a domiciliary of the state. Domicile is determined at the time the action is commenced.

Family members of a deceased individual properly filed an action for money damages in federal court against a funeral home for negligent infliction of emotional distress based on the alleged mishandling of the remains of the deceased. The funeral home timely and properly served an answer to the complaint. Thirty days later, including eight weekend days, the family members served a demand for a jury trial on the funeral home. The demand was served on a holiday. On the following day, a non-holiday weekday, the family members filed the jury trial demand with the court. Was the family members' demand for a jury trial timely?

A No, because the demand for a jury trial was not served on the funeral home within 14 days of service of the answer. BNo, because the demand for a jury trial was not filed with the court within 21 days of service of the answer. CYes, because the last day for making the demand was a holiday. DYes, because weekend days are not counted in determining whether the demand was timely. Answer choice A is correct. A demand for a jury trial must be served within 14 days after service of the last pleading directed to the issue that is sought to be tried by a jury. Here, the complaint sought damages, which is an action at law for which a jury trial may be had. However, the demand was not made within 14 days of service of the funeral home's answer, which is the only responsive pleading the funeral home may make that is directed to the plaintiff's complaint. Therefore, the demand for a jury trial was not timely.

An employer properly brought a conversion action against an employee in federal district court sitting in diversity jurisdiction. The employee's answer contained an age discrimination counterclaim against his employer based on federal law. The answer was prepared, signed, and filed by the employee's attorney. The employee's attorney did not act in bad faith, but did not reasonably investigate the factual contentions made in the counterclaim. The employer's attorney filed with the court and served on the employee's attorney a motion to dismiss the counterclaim for failure to state a claim upon which relief can be granted. The next day, the employer's attorney served a motion for sanctions under Rule 11 on the employee's attorney based on the failure to reasonably investigate the factual contentions contained in the counterclaim, but did not file this motion with the court. Twenty days later and prior to the court ruling on the dismissal motion, the employee's attorney filed a notice of dismissal with respect to the counterclaim. May the employer's attorney continue to pursue its motion for sanctions by filing it with the court?

A No, because the employee's attorney did not file the counterclaim in bad faith. BNo, because the employee's attorney filed a notice to dismiss the counterclaim. CYes, because the employee's attorney did not reasonably investigate the factual contentions contained in the counterclaim. DYes, because the employer's attorney served its motion on the employee's attorney prior to the dismissal of the counterclaim. Answer choice B is correct. A party (or party's attorney) may seek sanctions that can include the payment of reasonable attorney's fees and other costs attributable to violation of the certification made by a party or the party's attorney in presenting a pleading or other paper to the court. Among the certifications made is that factual contentions contained in a pleading have evidentiary support based on a reasonable inquiry. Although the employee's attorney violated this certification, the employee's attorney is not subject to sanctions because the attorney withdrew the counterclaim within 21 days of service of sanctions motion.

After two years of nonpayment of the installments of a mortgage loan by a mortgagor, a bank foreclosed on the mortgaged home pursuant to state law. The mortgagor owned and resided in the home and the bank had sent several requests for payment of the money owed under the mortgage to the mortgagor at his home address. The only notice of the foreclosure action provided to the mortgagor, however, was by publication in a local newspaper. A state statute permits "service by publication, mail, or personal service in any foreclosure action." The mortgagor does not subscribe to the local newspaper and never saw the newspaper notice. Following the foreclosure, the mortgagor filed a federal action under § 1983 alleging that foreclosure without actual notice effected a deprivation of property without due process. Is the notice by publication constitutionally sufficient?

A No, because the identity and address of the mortgagor was known to the bank. BNo, because the mortgagor never saw the notice in the newspaper. CYes, because state law provided for this method of notice. DYes, because the method of notice selected here was reasonably certain to inform parties affected by the action. Answer choice A is correct. Due process is met if the notice is reasonably calculated, under all the circumstances, to apprise interested parties of the pendency of an action and afford them an opportunity to present their objections. If the identity and address of an interested party are known or obtainable through reasonable efforts, then notice through in-person delivery, registered mail, return receipt requested, or some other means likely to notify the particular individual is required. Notice by publication under such circumstances would not be sufficient. Here, the bank knew the identity and address of the mortgagor, but only gave notice by publication. Such notice is not constitutionally sufficient.

In a civil action tried in federal district court, the judge determined that nine jurors were needed due to the anticipated length of the trial to ensure that six jurors remained when the case was sent to the jury for deliberation. The judge's determination was made in good faith and based on her experience as a judge dealing with juror requests for dismissal from a case. Consequently, nine individuals were selected as members of the jury. However, after each attorney had made his closing argument, all nine jurors remained. Without consulting either party, the judge, acting without discriminatory intent with regard to race or gender, excused the three jurors who had been selected last; she did not demonstrate good cause for excusing these jurors. Was this action proper?

A No, because the judge failed to consult either party. BNo, because the judge did not excuse the three jurors for good cause. CYes, because a verdict may be returned by a jury with as few as six members. DYes, because the judge acted without discriminatory intent with regard to the race or gender of the excused jurors. Answer choice B is correct. An individual who has been selected to serve as a juror must participate in the verdict unless excused for good cause, such as illness, family emergency, or misconduct. The federal rules do not authorize the dismissal of jurors to arrive at a specific number of jurors.

A patient brought a suit against a physician in federal district court under diversity jurisdiction, alleging medical malpractice based on state law. After the patient presented his case to the jury, the physician moved for judgment as a matter of law. In deciding the motion, the court reviewed all of the evidence presented by the patient in the light most favorable to the patient. The court considered the patient's only two witnesses to not be credible and gave no weight to their testimony. As a result, the court determined that the patient did not present sufficient evidence for a jury to reasonably return a verdict in the plaintiff's favor. Accordingly, the court granted the physician's motion for judgment as a matter of law. Was the court's ruling proper?

A No, because the motion cannot be made until both parties have been fully heard. BNo, because the court considered the credibility of the patient's witnesses and the weight of the evidence. CYes, because the court viewed the evidence in the light most favorable to the patient. DYes, because the court determined that patient's witnesses were not credible. Answer choice B is correct. In considering a motion for judgment as a matter of law, the court must view the evidence in the light most favorable to the opposing party and draw all reasonable inferences from the evidence in favor of the opposing party. It may not consider the credibility of witnesses or evaluate the weight of the evidence. A determination of the credibility of witnesses and the weight given to evidence must be made by the jury. Here, the court determined that the patient's two witnesses were not credible and gave no weight to their testimony in reviewing the motion for judgment as a matter of law. Thus, the court's ruling was improper.

The jury in a federal class action lawsuit rendered a verdict that awarded the plaintiffs substantial damages resulting from exposure to toxic chemicals released by the defendant corporation. The verdict was not against the clear weight of the evidence and the damages awarded were not excessive. Shortly after the entry of the judgment, the defendant timely filed a motion for a new trial based on newly discovered evidence. While this evidence existed at the time of the trial, the defendant was excusably ignorant of it until after the trial. Moreover, this evidence refuted evidence presented by the plaintiffs at trial that successfully undermined the corporation's defense. Should the court grant the corporation's motion for a new trial?

A No, because the verdict was not against the clear weight of the evidence. BNo, because the damages were not excessive. CYes, because of the newly discovered evidence. DYes, because the trier of fact was a jury rather than a judge. Answer choice C is correct. One ground for granting a new trial upon motion by a party is that newly discovered evidence that existed at the time of trial was excusably overlooked and would likely have altered the outcome of the trial.

A grocery store clerk found a ring while helping a customer with his groceries. Both the customer and the store owner have asserted a claim with regard to the ring, which has been valued at $67,000. The clerk, who has also claimed ownership of the ring, filed an interpleader action under Rule 22 in federal district court. The store owner and customer are citizens of the same state, and the clerk is a citizen of a neighboring state. The store owner has filed a motion to dismiss for lack of subject matter jurisdiction. Should the court grant the store owner's motion?

A No, because there is complete diversity between the clerk and the other two claimants. BNo, because the clerk has asserted a claim to the ring. CYes, because the customer and the store owner are citizens of the same state. DYes, because the value of the ring does not exceed $75,000. Answer choice D is correct. With regard to a federal interpleader action based on Rule 22, subject matter jurisdiction is determined in accord with the general rules for such jurisdiction. Here, because the value of the ring does not exceed $75,000, the amount-in-controversy requirement for diversity jurisdiction is not met and therefore the court lacks subject matter jurisdiction. Answer choice A is incorrect. Although there is diversity of citizenship between the stakeholder (the clerk) and the two other claimants, the value of the stake (the ring) is only $67,000. Consequently, the amount-in-controversy requirement is not met. Answer choice B is incorrect. Although under federal interpleader pursuant to Rule 22 the stakeholder may assert a claim to all or part of the stake, there is no requirement that the stakeholder do so. Answer choice C is incorrect. Under the federal interpleader rule, unlike the federal interpleader statute, diversity of citizenship is tested between the stakeholder and the other claimants. There is no requirement under the rule, unlike the statute, that there be diversity between at least two of the claimants.

A car driven by a vacationer and a tractor-trailer driven by truck driver collided with each other. The vacationer filed an action based on negligence against the truck driver in federal district court in the state in which he resided. The court had subject-matter jurisdiction over this action but not personal jurisdiction over the truck driver, who did not reside in the forum state but rather in the state in which the accident had occurred. The truck driver timely filed a motion to dismiss the action due to lack of personal jurisdiction and proper venue. May the court transfer the action to the federal district court for the district in which the truck driver resides if the court determines that such transfer is in the interest of justice?

A No, because venue is improper, the court must dismiss the action. B No, because the forum court lacks personal jurisdiction over the truck driver. C Yes, because transfer is permitted if the court has determined that the transfer is in the interest of justice. D Yes, even though venue is proper in the forum court. Answer choice C is correct. Although venue is improper in the forum court because the accident did not occur in the forum state nor did the defendant truck driver reside there, the forum court may transfer the action to any judicial district in which it originally could have been brought if it determines that such transfer is in the interest of justice. Here, the action could have been brought in the federal district court for the district in which the truck driver resides.

A plaintiff, a resident of State A, filed a complaint against two defendants in a state court in State B, where a car accident between the parties took place. The claims were based solely on state law. One defendant resides in State B, where the accident occurred, and another resides in State C, a neighboring state. The complaint alleged $100,000 in damages. The defendant who resides in State B was personally served with a complaint and summons while he was walking to work. The defendant who resides in State C was served by a process server in the forum state after attending a business meeting in that state. Shortly thereafter, both defendants filed a motion to remove the case to the federal district court in the district where the case was filed. The federal district court denied the motion. What is the most likely reason that the motion was denied?

A One of the defendants is a citizen of the state in which the action was filed. BThe federal court could not exercise personal jurisdiction over both defendants. CThe federal court lacked subject-matter jurisdiction over the claims. DVenue would be improper in the federal district court. Answer choice A is correct. A civil action filed in state court may be removed to the district court where the state court action commenced if that federal court has original jurisdiction over the matter. If removal is sought solely based on diversity jurisdiction, however, the claim cannot be removed if any defendant is a citizen of the state where the original action was filed.

A plaintiff filed a complaint in federal district court. The defendant was properly served with the complaint and summons and the process server properly filed a return of service with the court. The defendant has failed to timely serve an answer on the plaintiff or file it with the court. Which of the following steps should the plaintiff take first?

A Request the court clerk to enter a default. BFile a motion seeking sanctions under Rule 11. CFile a motion to strike under Rule 12. DFile a motion for a default judgment under Rule 55. Answer choice A is correct. When a defendant has failed to plead or otherwise defend an action, the plaintiff can request that the court clerk enter a default. The court clerk must enter a default if the plaintiff can establish the defendant's failure, usually by affidavit.

The defendant lives in State A. He commutes to work in nearby State B, where he does most of his shopping (State B has no sales tax). The defendant spends a month each summer vacationing in State C. While driving from home to State C, Defendant traveled through State D for the first time, where he negligently hit and totaled the $100,000 sports car of the plaintiff, a State D citizen. The defendant continued on to State C, where he vacationed for several weeks. The day after the accident, the plaintiff filed a complaint in federal court and sought to serve process on the defendant the next day, either personally or by mail. States A, B, C, and D all have statutes authorizing personal jurisdiction over anyone who engages in activity within the state, to the extent permissible under the U.S. Constitution. Assuming subject-matter jurisdiction is proper, in which state is the plaintiff least likely to find personal jurisdiction over the defendant?

A State A. BState B. CState C. DState D. Answer choice B is correct. Defendant is neither domiciled not present in State B, so on the day after the accident, State B would have no personal jurisdiction over him. Answer A is incorrect because personal jurisdiction exists in the state in which the defendant is domiciled, which is likely State A, the state in which he lives. Answer C is incorrect because the defendant is voluntarily present in State C and may be served with process there.

A plaintiff filed suit against a defendant in a federal district court sitting in diversity jurisdiction, alleging state law fraud. In discovery, the plaintiff served 32 interrogatories on the defendant's bank for information regarding the defendant's banking transactions related to the subject matter of the lawsuit. How can the bank respond to the interrogatories?

A The bank may refuse to answer interrogatories 26 through 32 because they exceed the numerical limitation on interrogatories. BThe bank may refuse to answer all of the interrogatories because the plaintiff may only serve 25 interrogatories. CThe bank must move for a protective order to avoid answering the plaintiff's interrogatories. DThe bank has no obligation to answer any of the interrogatories that have been served by the plaintiff. Answer choice D is correct. Interrogatories may only be served on a party to a lawsuit; they may not be used on nonparty witnesses. Here, the defendant's bank is not a party to the litigation and therefore has no obligation to answer any of the interrogatories served by the plaintiff.

A celebrity with diminished capacity and a filmmaker entered into a written contract for the filmmaker to create a documentary of the celebrity's life. The contract obligated the celebrity to pay $100,000 if the celebrity was satisfied with the film. The filmmaker created the film, but after previewing the film, the celebrity refused to pay. The filmmaker filed an action for breach of contract in federal district court. Although the filmmaker's complaint was sufficient with regard to jurisdiction, it did not allege with specificity that (a) the celebrity had the capacity to contract, (b) the condition precedent to the celebrity's performance under the contract had been satisfied, and (c) there was not a mutual mistake as to the currency in which payment was to be made. The complaint also stated that (d) the filmmaker was entitled to special damages. The celebrity filed a motion for a more definite statement regarding each of these items. With respect to which of these items is a court most likely to require greater specificity from the filmmaker?

A The celebrity's capacity to enter into the contract. BThe satisfaction of the condition precedent to the celebrity's performance. CThe absence of mistake as to the currency in which payment was to be made. DThe special damages sought by the filmmaker. Answer choice D is correct. When a plaintiff seeks special damages, the plaintiff is required to plead such damages with specificity.

A plaintiff brought a civil action in federal district court in the district of her residence for violation of a federal antitrust statute against several defendants, two of whom lived out of state. To establish a statutory violation, the plaintiff must prove that the defendants entered into an illegal agreement. The complaint does not contain sufficient facts to establish that the defendants had entered into such an agreement, but does contain enough facts to raise a reasonable expectation that discovery will reveal evidence of such an agreement. The court, however, believes that it is more likely than not that the plaintiff will be unable to find such evidence, and hence, it is probable that the action will be unsuccessful. Prior to filing their answer, the out-of-state defendants moved to dismiss the action. If the court grants the defendants' motion, what is the most likely reason?

A The complaint fails to contain sufficient facts that the defendants had entered into an illegal agreement. BIt is more likely than not that the plaintiff will be unable to find evidence of an illegal agreement. CIt is probable that the action will be unsuccessful. DThe plaintiff brought the action in the wrong district. Answer choice D is correct. If the plaintiff has brought the case in the wrong district, then the court may dismiss for improper venue or transfer the case to any district where the case might properly have been brought. While it is certainly more likely that the court would transfer the case, it is legally authorized to dismiss it. Note that, to reach this answer choice, you don't have enough information to definitely say that venue is improper; however, because the other three answer choices are incorrect, D must be correct as it is a valid basis for granting a motion to dismiss.

The spouse of a deceased properly brought a wrongful death action in federal district court based on diversity jurisdiction. The defendant then properly impleaded a third party, alleging that the negligence of the third party was the cause of the deceased's death and therefore the third party must indemnify the defendant if the defendant were found liable to the spouse. The spouse sought leave to amend her complaint to include a negligence claim against the third party. The third party, noting that both the spouse and the third party are citizens of the same state, has challenged the amendment on grounds of lack of subject matter jurisdiction. How should the court respond to the third party's challenge?

A The court must deny the spouse leave to amend because the court lacks both diversity and supplemental jurisdiction over the claim against the third party. BThe court may exercise its supplemental jurisdiction and permit the spouse to amend her complaint. CThe court must permit the spouse to amend her complaint because the original complaint was properly brought in federal district court. DThe court must permit the spouse to amend her complaint because the defendant properly impleaded the third party. Answer choice A is correct. Subject matter jurisdiction does not exist with respect to the spouse's claim against the third party based on diversity jurisdiction because both the spouse and the third party are citizens of the same state. In addition, in a federal court action based solely on diversity jurisdiction, supplemental jurisdiction is precluded with respect to a claim by an existing plaintiff against a person who is made a party to the action through impleader.

A state legislature enacted a statute providing that punitive damages were recoverable against an employer in a claim filed by an employee only if the employee waived the right to receive unemployment benefits. After the statute was enacted, many employees challenged the law and the state's lower appellate courts all ruled that the statute was valid under state constitutional law. A claim was then filed in federal district court in the state in which the statute had been enacted. The state supreme court had not yet addressed the constitutional issue. When determining whether to award punitive damages to the employee who had brought the claim, the federal district court followed the state court's intermediate appellate rulings. After the federal court's decision, but before a final appeal in the federal-court matter had been disposed of by the court of appeals, the state supreme court held that the statute was unconstitutional under state law. Which of the following is a correct statement?

A The court of appeals must apply what it believes is the proper legal interpretation of state law on the issue. BThe court of appeals must defer to the ruling of the district court that followed the state court's intermediate appellate rulings on the issue. CThe court of appeals must apply the state supreme court's ruling on the issue. DThe court of appeals may, but need not, follow the state supreme court's ruling on the issue. Answer choice C is correct. A federal appellate court is bound by the decision of a state's highest court on an issue even if it is handed down after a federal district court action on that issue was completed. The federal appeals court is bound by that ruling, so long as the final appeal has not been disposed of in the matter.

A plaintiff sued two defendants in federal district court, one for breach of contract and the second for tortious interference with a contractual relationship. The contract defendant filed a motion to dismiss for lack of subject matter jurisdiction. The court determined that, while the court did lack subject matter jurisdiction, dismissal of this defendant would, as a practical matter, impair the defendant's ability to protect its interests and that the contract defendant was an indispensable party. What action should the court take?

A The court should grant the motion to dismiss the claim against the contract defendant and dismiss the action because the contract defendant is an indispensable party. BThe court should deny the motion to dismiss because the contract defendant is a necessary party. CThe court should deny the motion to dismiss because the contract defendant is an indispensable party. DThe court should deny the motion to dismiss because the contract defendant is both a necessary party and an indispensable party. Answer choice A is correct. The contract defendant is a necessary party whose joinder is required because dismissal of this defendant would, as a practical matter, impair the defendant's ability to protect its interests. However, because the court must have subject matter jurisdiction over a party, the court must dismiss the contract defendant as a party to this action. Having done so and also determined that the dismissed party is an indispensable party, the court must dismiss the action.

The owner of a patent for a medical device initiated an action in federal district court against a defendant. The complaint alleged that the defendant had manufactured a device that infringed on the plaintiff's patent. One type of relief sought by the plaintiff was a preliminary injunction. What must the court find in order to grant the preliminary injunction requested by the plaintiff?

A The defendant has infringed on the plaintiff's patent beyond a reasonable doubt. BThe plaintiff has presented plausible evidence of patent infringement by the defendant. CThe plaintiff is likely to succeed on the merits of her patent infringement action at trial. DThe plaintiff has presented clear and convincing evidence of patent infringement by the defendant. Answer choice C is correct. To grant a preliminary injunction, the court must determine that the plaintiff is likely to succeed on the merits of his action at trial.

The spouse of a deceased person filed a complaint in federal district court based on diversity jurisdiction. The complaint alleged that the deceased had died six months previously from a defective product manufactured by the defendant. The spouse took no action with regard to serving the complaint on the defendant. Nine months after the complaint was filed, the court notified the spouse that the complaint would be dismissed unless the spouse established good cause for not timely serving the complaint and summons. The spouse failed to appear in court and did not establish good cause for not serving the complaint and summons. The court dismissed the action. Is the court's dismissal with or without prejudice?

A The dismissal is with prejudice because the spouse failed to appear in court to establish good cause for not serving the complaint. B The dismissal is with prejudice because more than 120 days had passed since the complaint was filed. C The dismissal is without prejudice, because process was not timely served. D The dismissal is without prejudice, because the court acted sua sponte to dismiss the complaint. Answer choice C is correct. When an action is dismissed due to the failure to timely serve process, the dismissal is without prejudice.

A parking garage attendant found a necklace beside a car in the garage. Both the owner of the car and the owner of the garage claim ownership of the necklace, which has been valued at $73,000. The car owner has filed an action to gain possession of the necklace in state court naming the attendant as defendant. The attendant filed a federal statutory interpleader action in federal district court. The garage owner and the car owner are citizens of the same state and the attendant is a citizen of a neighboring state. The attendant has posted a bond with the federal court, but retains possession of the necklace. The car owner has filed a motion to dismiss the interpleader action for lack of subject matter jurisdiction. For which of the following reasons should the court grant the car owner's motion?

A The garage owner and the car owner are citizens of the same state. BThe value of the necklace does not exceed $75,000. CThe attendant has retained possession of the necklace. DThe car owner had already filed an action in state court. Answer choice A is correct. Under the federal interpleader statute, diversity jurisdiction is met if any two claimants are citizens of different states. Here, the two claimants, the garage owner and the car owner, are both citizens of the same state. Therefore, the court lacks subject matter jurisdiction over the action.

A party properly filed suit in federal district court in State A against a corporation that was incorporated in State B. The corporation successfully transferred the action to a federal district court in State B under the venue rules of 28 U.S.C. § 1404 for the convenience of the parties and witnesses. Which state's conflict-of-law rules should the district court in State B apply?

A The laws of State A, because it is the transferor court. B The laws of State B, because it is the transferee court. C The court may use its discretion to determine which laws to apply. D The parties may agree to use the law of either state. Answer choice A is correct. If venue is transferred under the change of venue rules of 28 U.S.C. § 1404, the new court must apply the law of the state of the transferor court, including that state's rules regarding conflict of laws.

A traffic accident victim initiated a diversity action for damages stemming from an accident against a truck driver and the corporation who had hired him as an independent contractor for a one-time delivery, to fill in for their own truck driver, who had fallen ill. The victim filed the complaint, which was based on negligence with regard to the truck driver and respondeat superior with respect to the corporation, in the federal district court for State A, the state in which the accident occurred. The truck driver was transporting equipment from State B, the state of incorporation and principal place of business of the corporation, to State C, where the corporation was opening a new plant. The corporation had no business dealings in State A. The victim is a citizen of State D. The forum state has a long-arm statute that permits a state court to exercise personal jurisdiction to the extent permitted by the Due Process Clause of the Fourteenth Amendment to the U.S. Constitution. What is the corporation's best argument that the court lacks personal jurisdiction over it?

A The minimum contacts test cannot not be satisfied due to the corporation's lack of business dealings in State A. BThe victim is not a citizen of State A. CThe corporation had not purposefully availed itself of the benefits and laws of State A. DThe truck driver was an independent contractor, rather than an employee of the corporation. Answer choice D is correct. In order for a defendant to be subject to the jurisdiction of a court, the defendant must have sufficient minimum contacts with the forum state. While a single contact can be sufficient if the cause of action is based on that contact, an out-of-state corporation, is not subject to personal jurisdiction solely because of contacts in the state by an independent contractor. Contacts by a nonresident employer's agents or employees, on the other hand, are generally imputed to the employer when the agent or employee is acting within the scope of the agency or employment.

During the process of jury selection in a civil action filed in federal district court, the plaintiff exercised each of his permitted peremptory challenges to exclude females from the jury. The defendant objected to the plaintiff's exercise of his peremptory challenges. The plaintiff explained that, because he was male, females were more likely than males to be biased against him. For whom should the court rule with regard to the defendant's objection?

A The plaintiff, because the plaintiff's challenges were not made for a racial-based reason. BThe plaintiff, because a peremptory challenge may be exercised without stating a reason. CThe defendant, because the plaintiff's challenges were exercised for the purpose of excluding females. DThe defendant, because the plaintiff excluded females with each of his permitted peremptory challenges. Answer choice C is correct. While peremptory challenges may generally be made for any reason, such challenges may not be made for a gender-based reason. Here, the plaintiff's explanation for making his challenges was based on the gender of the prospective jurors. Therefore, the peremptory challenges were not proper and the court should rule in favor of the defendant.

A group of patients sought certification as a class in federal district court for a class-action lawsuit against a nursing home for damages resulting from substandard nursing care. The district court granted certification as a class, and the defendant nursing home timely and properly filed a petition for permission to appeal the certification with the appropriate court of appeals. If the court of appeals permits the nursing home's appeal, what is the effect on the district court proceedings?

A The proceedings in the district court are automatically stayed pending the appeal. BThe proceedings in the district court are stayed pending the appeal if the district court or the court of appeals so orders. CThe proceedings in the district court cannot be stayed pending the appeal. DThe proceedings in the district court can only be stayed pending the appeal by order of the district court. Answer choice B is correct. If an appeal is permitted, it will not stay proceedings in the district court unless the district court or the court of appeals so orders.

A plaintiff initiated a negligence suit in federal district court against a defendant based on an automobile accident. The accident occurred as the defendant, tired from a long trip, was driving through a state to reach his home in a neighboring state. The forum court, in a state with two federal districts, was located in the eastern district. The accident occurred in the western district of this state. The defendant, who is a citizen of a foreign country but is also a lawful, permanent resident of the United States, filed a timely motion to dismiss the action for lack of subject-matter jurisdiction. The court properly rejected the motion. In his answer, the defendant raised the defense of improper venue. The plaintiff contemplates making the following replies; which correctly reflects the law?

A Venue is proper because an alien may be sued in any judicial district. BVenue is proper because the accident occurred in the forum state. CBecause the court has ruled that it has subject-matter jurisdiction over the suit, venue is also proper. DThe defendant waived this defense by failing to assert it in his motion to dismiss. Answer choice D is correct. An objection to venue is waived by a defendant if not asserted in a pre-answer motion to dismiss if the defendant chooses to file one. Here, because the defendant failed to assert this objection in his motion to dismiss, he has waived his objection to the improper venue

A condominium association filed an action under a federal statute in federal district court against the original developer of the condominium. The association's complaint sought to void an agreement between the association and the developer that gave the developer the right to use an office in the condominium building rent free. The complaint also contained a demand for a jury trial. The developer timely filed a motion to strike this demand. The federal statute that created this cause of action is silent as whether to an action filed under it may be heard by a jury. Which standard should the court apply in ruling on this demand?

A Whether the law of the state in which the federal district court sits permits a jury trial with regard to the relief sought. BWhether there is a genuine dispute as to a material fact. CWhether the issue to be determined is primarily one of fact or one of law. DWhether the relief sought is equitable or legal in nature. Answer choice D is correct. If a new cause of action that was unknown at common law is created, such as the statutory cause of action here, then the court must look to whether the remedy sought is legal rather than equitable; if it is legal, a jury should be allowed.

A patron who was being seated at a restaurant sustained injuries when another patron, agitated over not being seated, started a physical altercation. The patron filed a complaint in federal district court against both the other patron and the restaurant, alleging that the assault and battery committed by the other patron and the negligence of the restaurant resulted in the patron's injuries. The patron's complaint asserted that the other patron and the restaurant were jointly and severally liable for his injuries. The complaint did not allege that the defendants acted in concert. Is joinder of these defendants and claims proper?

A Yes, because a plaintiff may bring whatever claims he has, including independent claims, against any party. BYes, because the defendants were alleged to be jointly and severally liable for the patron's injuries. CNo, because the complaint did not allege that the defendants acted in concert. DNo, because the same cause of action was not pled against each defendant. Answer choice B is correct. In order to join defendants in a single action, the plaintiff's right to relief must be asserted against them jointly, severally, or in the alternative with respect to or arising out of the same transaction or occurrence. Here, the patron has asserted that the other patron and the restaurant are jointly and severally liable for his injuries that arose out of the physical altercation started by the other patron.

The owner of an airplane sued two buyers, a father and his adult son, in federal district court for breach of contract to purchase the airplane. The father is a citizen of a foreign country, but is a lawful permanent resident of the United States. His son is a citizen of the United States. Both the owner of the airplane and the father reside in the middle judicial district of a state with three federal judicial districts. The son resides in the eastern judicial district of that state. The airplane is located in the western judicial district, which is where the negotiations to purchase the airplane and the execution of the contract took place. The forum court is located in the middle judicial district. The son has timely filed a motion to dismiss the action due to improper venue. Should the court deny this motion?

A Yes, because an alien may be sued in any judicial district in the United States. BYes, because the father resides in the forum court's district and his son resides in the same state. CNo, because the contract was negotiated and signed in the western district and the airplane is located there. DNo, because the son does not reside in the forum court's district. Answer choice B is correct. Venue is proper in a judicial district in which any defendant resides, if all defendants reside in the same state in which the district is located. An alien who is a lawful, permanent resident of the United States is treated as a United States citizen for venue purposes. Venue based on residency is determined by the judicial district in which such an alien resides. Consequently, venue is proper in this case because the father resides in the middle judicial district, the forum court's district, and his son resides in the eastern district of the same state.

An attorney filed a complaint in federal court, alleging that his client's property had been damaged through vandalism perpetrated by the defendant. The attorney was reasonably certain that evidence would support that the vandalism had been perpetrated by the defendant, but he did not have clear evidence at the time of the submission. He was confident that the discovery process would lead to the identification of the correct defendant and knew that if the defendant could not be identified, the claim would be unsuccessful. He did note on the pleading that he intended to learn such evidence during the discovery process. Is the attorney subject to Rule 11 sanctions based on the filing?

A Yes, because he knew the factual allegations were inaccurate. B Yes, because he is relying on discovery to establish an essential element of the claim. C No, because he believed he would discover the correct defendant after discovery. D No, because discovery had not yet commenced. Answer choice C is correct. An attorney submitting a pleading to the court is certifying that to the best of her knowledge, information, and belief, the factual contentions have evidentiary support or, if specifically so identified, will likely have evidentiary support after a reasonable opportunity for further investigation or discovery. Here, although the attorney does not have the necessary evidence, he does expect to discover it shortly.

A plaintiff filed a civil action against a defendant in federal district court on July 1. On July 6, before the defendant filed an answer, the plaintiff served him with an amended claim that included causes of action that arose after the date of the initial filing. The court ordered the defendant to respond by July 15. On July 18, the defendant filed an answer to the amended complaint. Has the defendant filed a timely answer?

A Yes, because he responded within 14 days of the service of the amended pleading. B Yes, because the response time began running on July 6. C No, because the court ordered that he respond by July 15. D No, because the original filing occurred on July 1. Answer choice C is correct. Unless the court orders otherwise, a party must respond to an amended pleading within the later of 14 days after service of the amended pleading or the time remaining for response to the original pleading. Here, the defendant only had until July 15 because the court ordered that he respond by that date. Answer choice A is incorrect because the court can order a responsive time earlier or later than would typically be required. If it does, the court-ordered time period applies. RULE: W/IN 14 DAYS AFTER SERVICE OF AMENDED PLEADING, OR TIME LEFT ON ORIGINAL PLEADING, WHICHEVER IS LATER; UNLESS COURT GIVES SPECIFIC TIME.

A plaintiff filed an action in federal district court based on a state law claim. The court had diversity jurisdiction over the action. An entity timely sought to intervene as of right, contending that, although no federal statute conferred this right, the entity's interest was not adequately represented by the existing parties. Although it found that the entity had the right to intervene, the court refused to permit the intervention because, even though the entity's presence in the action would not violate the diversity-of-citizenship requirement, the entity's claim did not satisfy the amount-in-controversy requirement. Was the court's refusal proper?

A Yes, because intervention is not permitted in an action over which the court has diversity jurisdiction. BYes, because the court did not have subject matter jurisdiction over the entity's claim. CNo, because the entity's intervention was as of right rather than permissive. DNo, because the court had supplemental jurisdiction over the entity's claim. Answer choice B is correct. Even when a person seeks to intervene as of right in an action for which subject matter jurisdiction is based solely on diversity jurisdiction, the court must have subject matter jurisdiction over the intervenor's claim. When the court does not have subject matter jurisdiction over a claim, the court should not permit the person to intervene.

A plaintiff was severely injured when he slipped while shopping at the defendant's department store. As a result, the plaintiff brought a state law negligence action against the defendant in federal district court under diversity jurisdiction. The defendant was properly served on January 1 and filed his answer on January 15. Discovery for the matter concluded on June 1. On June 25, three weeks prior to trial, the plaintiff filed a motion for summary judgment. Was the plaintiff's motion for summary judgment timely?

A Yes, because it was filed within 30 days of trial. BYes, because it was filed within 30 days after the close of all discovery. CNo, because the motion must be filed prior to the close of all discovery. DNo, because the motion must be filed within 15 days after the close of all discovery. Answer choice B is correct. A party may file a motion for summary judgment at any time until 30 days after the close of all discovery. Here, discovery closed on June 1 and the plaintiff filed a motion for summary judgment on June 25. Therefore, the motion for summary judgment was timely.

The owner of a car was severely injured and her car was destroyed when she became distracted by a display on an electronic billboard and lost control of her car. The car owner brought a negligence action based on diversity jurisdiction in federal district court against the owner of the billboard, as well as the advertising firm that created the display and the manufacturing company that sold the product displayed. The manufacturing company filed a cross-claim against the advertising firm based on an indemnification clause in their contract. Instead of filing an answer, the advertising firm served a motion to dismiss the cross-claim for failure to state a claim upon which relief can be granted. An affidavit regarding the oral waiver of the indemnification clause was attached to the motion. Before the court takes any action with regard to the advertising firm's motion, can the manufacturing company voluntarily withdraw the cross-claim without the approval of the court or the consent of the parties?

A Yes, because the advertising firm has not served an answer to the cross-claim B Yes, because a cross-claim is never compulsory. C No, because the advertising firm's motion to dismiss was converted into a motion for summary judgment. D No, because the advertising firm served a responsive pleading. Answer choice A is correct. A party may voluntarily dismiss a cross-claim (or a counterclaim or third-party claim) without the approval of the court or the consent of the parties before a responsive pleading is served, or if there is no responsive pleading, before evidence is introduced at a hearing or trial. The service of a summary judgment motion by a co-party with respect to a cross-claim (or counterclaim or third-party claim) does not cut off a party's ability to voluntarily dismiss the claim without court approval or the consent of the other parties. This occurs only after evidence is introduced at the court hearing on the motion. Here, although the motion filed by the advertising firm was a motion to dismiss for failure to state a claim upon which relief can be granted, the attachment of an affidavit presenting evidence beyond the pleadings converted the motion into a summary judgment motion. Nevertheless, because the court had not held a hearing on this motion at which evidence was presented, the manufacturing company can voluntarily withdraw the cross-claim without court approval or the consent of the parties.

A bakery negotiated with a supplier regarding a long-term commitment to supply flour. They agreed to specific terms to be memorialized in a written contract. Due to an oversight, the supplier never signed the written contract, although it began providing flour to the bakery under the terms of their agreement. Six months later, the price of flour rose dramatically following a nationwide drought. The supplier, realizing that it had never signed the contract, told the bakery that it would have to charge a higher price for the flour. The bakery properly filed a complaint in federal district court, alleging that the flour supplier had breached their contract. The flour supplier filed an answer in which it denied the factual allegations in the complaint regarding the price and duration of the contract, but did not raise any affirmative defenses. The following day, the flour supplier filed a motion to dismiss, asserting that the oral contract between the parties violated the Statute of Frauds. Can the court grant the defendant's motion to dismiss?

A Yes, because the answer denied the factual allegations regarding the terms of the contract. B Yes, because the complaint failed to state a claim upon which relief could be granted. C No, because the defendant waived any objection on the pleadings by filing an answer. D No, because the defendant did not assert a defense based on the Statue of Frauds in its answer. Answer choice D is correct. An answer must not only admit or deny the allegations of the plaintiff's complaint but also state applicable affirmative defenses, including the Statute of Frauds. Those affirmative defenses not stated in the defendant's answer are deemed waived. In this case, the defendant did not assert the affirmative defense of Statute of Frauds, and thus the defense would be deemed waived.

A plaintiff filed a complaint in federal district court against a defendant, a natural person, based on diversity jurisdiction. Two days later, the plaintiff sent a notification of the action and a proper request for waiver of service of process to the defendant, who was domiciled in another state. Upon receiving the request, the defendant signed and returned the waiver. Thirty days after the plaintiff sent the request for waiver of service and 20 days after the defendant returned the signed waiver, the defendant served his answer on the plaintiff. Was the defendant's answer timely?

A Yes, because the answer was served within 21 days after the defendant returned the waiver of service. B Yes, because the answer was served within 60 days after the plaintiff sent the request for waiver of service. C No, because the answer was not served within 14 days after the defendant received the request for waiver of service. D No, because the answer was not served within 30 days after the complaint was filed. Answer choice B is correct. If a defendant timely returns a waiver of service before being served with process, then the defendant does not have to serve an answer to the complaint until 60 days after the request was sent, or 90 days after it was sent to a defendant outside a judicial district of the United States.

A plaintiff filed an action against a defendant in federal district court. The complaint alleged that the defendant had infringed upon a trademark held by the plaintiff under federal law and sought $55,000 in damages. In addition, the plaintiff claimed damages of $30,000 allegedly attributable to the defendant's negligence in causing an auto accident involving the two parties. The plaintiff and the defendant are citizens of different states. The defendant moved to dismiss the negligence claim for lack of subject matter jurisdiction. The court denied the motion. Is the court's ruling correct?

A Yes, because the court can exercise supplemental jurisdiction over the negligence claim. BYes, because the court has diversity jurisdiction over the negligence claim. CNo, because neither diversity nor supplemental jurisdiction exists with respect to the negligence claim. DNo, because a state law based claim may not be joined with a claim over which the court has federal question jurisdiction. Answer choice C is correct. While the court has subject matter jurisdiction over the trademark infringement claim based on federal question jurisdiction, the court does not have subject matter jurisdiction over the negligence claim. The negligence claim is based on state, not federal law; therefore, the court does not have federal question jurisdiction over the claim. In addition, although the diversity-of-citizenship requirement is satisfied because the parties are citizens of different states, the amount-in-controversy requirement is not satisfied because the plaintiff is seeking damages of only $30,000. The negligence claim does not arise out of the same "common nucleus of operative fact" as the trademark infringement claim, so supplemental jurisdiction over the negligence claim would also not be appropriate. Consequently, the court should grant the defendant's motion to dismiss the negligence claim.

An account holder designated the plaintiff as the payee for an account held at a state bank. Upon the account holder's death, the plaintiff brought an action in federal court to compel the bank to pay to him the $95,000 held in the account. The bank balked at paying the plaintiff because a second claimant had informed the institution that the accountholder, prior to her death, had transferred her rights to the account to the second claimant. The plaintiff filed the action in a federal district court located in the state of which both the plaintiff and the second claimant are citizens. The bank is a citizen of another state. The bank has filed a motion to dismiss for failure to join a necessary party. Is the second claimant a necessary party?

A Yes, because the court has personal jurisdiction over the second claimant. BYes, because the bank could have to pay the second claimant as well as the plaintiff the amount in the account. CNo, because the bank can file a counterclaim and interplead the second claimant. DNo, because the joinder of the second claimant would defeat diversity jurisdiction. Answer choice B is correct. A party is considered necessary if the party's absence would leave the existing parties subject to a substantial risk of multiple or inconsistent obligations.

An employer filed suit in federal district court against a district director of the IRS for a refund of federal unemployment taxes that were allegedly wrongfully collected by the director under color of her legal authority. The employer now resides in the forum state, which has only one federal judicial district. For business reasons, the employer has recently moved from the state in which all the relevant events on which the claim is based took place and in which the director resides. The director has timely filed a motion to dismiss the action for improper venue. Should the court grant this motion?

A Yes, because the director resides in a state other than the forum state. BYes, because all of the relevant events on which the claim is based took place in a state other than the forum state. CNo, because the director was acting under color of her legal authority. DNo, because the court must transfer the case to the state in which all of the relevant events on which the claim is based took place. Answer choice C is correct. An action against a federal officer or employee acting in an official capacity or under color of legal authority may be brought in the judicial district in which the plaintiff resides.

An employee filed an action in federal district court against her employer, who does business as a sole proprietor, for violation of federal law regarding overtime pay. Both the employee and her employer reside within the geographic boundary of the district of the court where the action was filed. The business is located in the same state, but within the geographic boundaries of another federal district court. The employer has timely filed a motion to dismiss the action due to improper venue. Can the court grant this motion?

A Yes, because the events or omission on which the action is based took place in a judicial district other than the district in which the action was filed. BYes, because the court lacks diversity jurisdiction. CNo, because the employee resides in the district of the court where the action was filed. DNo, because the employer resides in the district of the court where the action was filed. Answer choice D is correct. Venue is proper in the judicial district in which the defendant resides. Because this action was filed in that district, the venue is proper and the court should deny the motion.

A farmer's property was damaged when a factory from a nearby state dumped hazardous waste on it. The farmer properly brought suit for negligence against the factory in federal district court based on diversity jurisdiction. Throughout discovery, the factory executives intentionally delayed the process and regularly refused to provide relevant documentation requested by the farmer's attorney, even though they had the documents and had no valid legal reason not to produce them. After a motion to compel by the farmer, the factory executives did eventually provide the evidence a day before trial. The farmer was ultimately successful in his claim, though the delays resulted in far higher litigation costs than would have occurred if the factory executives had produced the evidence in a timely fashion. Despite the fact that the farmer did not request attorney's fees, and the state in which the district court was located did not allow an award of attorney's fees, the court, on its own initiative, awarded them to the farmer. Did the court err by awarding attorney's fees to the farmer?

A Yes, because the forum state did not allow the award of attorney's fees. BYes, because the farmer did not request attorney's fees. CNo, because the factory executives acted in bad faith. DNo, because the defendant did not provide the requested evidence in a timely fashion. Answer choice C is correct. In a diversity case on a state law claim, the federal court may properly use its inherent power to assess attorney's fees as a sanction for a defendant's bad-faith conduct during the litigation, even if the law of the forum state provides that attorney's fees may not be awarded to a successful party.

A plaintiff brought suit against a defendant in federal district court alleging that the defendant violated the Federal Credit Card Protection Act, which creates a private cause of action. The plaintiff also filed a claim against the defendant for state-law fraud and theft. The claims stem from an incident in which the defendant allegedly took the plaintiff's credit card from a restaurant table and made $100,000 worth of purchases using the credit card. If the defendant moves to dismiss the fraud and theft claims due to lack of jurisdiction, will he prevail?

A Yes, because the fraud and theft claims do not present a federal question. BYes, because the claims stem from the alleged theft of the plaintiff's credit card, which is governed by state law. CNo, because the alleged damages satisfy the amount-in-controversy requirement. DNo, because the fraud and theft claims and the Federal Credit Card Protection Act claim arise from a common nucleus of operative fact. Answer choice D is correct. A district court with jurisdiction may exercise supplemental jurisdiction over additional claims over which the court would not independently have subject-matter jurisdiction but that arise out of a common nucleus of operative fact.

A homeowner who suffered the destruction of her home by fire properly filed a products liability action against the manufacturer of her toaster in federal district court. Under the applicable state law, a product defect must be proven in order to establish a products liability cause of action. Mere proof of a malfunction is not sufficient by itself to establish a product defect. At the close of the homeowner's presentation of her case to the jury, the manufacturer moved for judgment as a matter of law because the homeowner had failed to establish that there was a defect with respect to the toaster. The homeowner opposed this motion. The court determined, viewing the evidence in the light most favorable to the homeowner, that the homeowner had established by a preponderance of the evidence both that the toaster was the source of the fire and that the toaster had malfunctioned. Should the court grant the manufacturer's motion?

A Yes, because the homeowner failed to prove that there was a defect with respect to the toaster. B Yes, because the court has broad discretion in deciding whether or not to grant a motion for a judgment as a matter of law. C No, because, viewing the evidence in the light most favorable to the homeowner, she established that the toaster had malfunctioned. D No, because judgment as a matter of law cannot be granted until the defendant concludes the presentation of its case to the jury. Answer choice A is correct. Under Rule 50(a), once a party has been fully heard on an issue at a jury trial, the court may grant a motion for judgment as a matter of law resolving the issue against a party if the court finds that there is insufficient evidence for a jury reasonably to find for that party. Because the homeowner failed to present sufficient evidence to establish that there was a defect with respect to the toaster, which is a necessary element of a products liability action under the applicable state law, the court should grant the manufacturer's motion for a judgment as a matter of law

A plaintiff filed a complaint in federal district court based on diversity jurisdiction. The defendant operated a car repair business as a sole proprietorship. The complaint alleged that, due to the defendant's negligence in repairing the plaintiff's car, the plaintiff was involved in an accident and suffered over $100,000 in damages. The plaintiff's lawyer served the complaint and summons on the manager of the business at the defendant's car repair garage. Unlike the federal rules, the procedural rules of the forum state permit service of process at the defendant's regular place of business on a person of suitable age and discretion. The defendant timely filed a motion to dismiss based on insufficient service of process. The court denied this motion. Was the court's denial of the defendant's motion proper?

A Yes, because the law of the forum state permits service of process at the defendant's place of business. BYes, because the complaint alleged that the plaintiff had suffered damages in excess of the required amount in controversy. CNo, because process was served by the plaintiff's lawyer. DNo, because the federal rules do not permit this manner of service. Answer choice A is correct. Although the federal rules do not specify that service of process is permitted at the defendant's regular place of business on a person of suitable age and discretion, the federal rules do provide that service of process may be made by following the law of the forum state for service of process. Answer choice C is incorrect because service of process may be made by any nonparty who is at least 18 years old. (Note that an attorney may be disqualified from representing the plaintiff if the attorney is required to testify as to the propriety of the service of process, though this does not affect the validity of the service.)

A plaintiff sued a defendant homeowner in federal district court under diversity jurisdiction, alleging negligence arising from falling over a torn carpet at the defendant's home at a party at which the plaintiff was an invited guest. The plaintiff has asserted that the defendant should have discovered the tear in the carpet and fixed it before the party. After the plaintiff filed his claim, but before the trial on the merits began, the forum state's highest court held that a social guest is a licensee, not an invitee, and is not owed a duty of reasonable inspection by a property owner. The federal district court judge overseeing the case strongly disagrees with this ruling and has learned that the modern trend, adopted in many states, is to require a duty of reasonable inspection for both licensees and invitees. Can the federal court properly impose a duty of reasonable inspection on the homeowner?

A Yes, because the matter is being heard in federal court, rather than state court. B Yes, because the matter is substantive, rather than procedural. C No, because it is bound by the ruling of the forum state's highest court. D No, because the decision from the state's highest court came out after the plaintiff filed suit in the current case. C

The plaintiff, a resident of State A, was involved in an automobile accident with the defendant, who is a resident of State B. The plaintiff filed a negligence claim against the defendant in federal district court seeking to obtain $80,000 in damages. At the time of the accident, a passenger was riding in the plaintiff's car. The passenger, a resident of State B, also suffered injuries and seeks to join in the plaintiff's claim alleging $50,000 in damages. Does the federal district court have jurisdiction over the passenger's claim?

A Yes, because the passenger's claim and the plaintiff's claim arise out of a common nucleus of operative fact. BYes, because an additional claim need not meet the amount-in-controversy requirement for federal diversity jurisdiction. CNo, because allowing joinder of the passenger's claim would destroy complete diversity. DNo, because the passenger's claim does not meet the amount-in-controversy requirement for federal diversity jurisdiction. Answer choice C is correct. Although a federal district court with jurisdiction over a claim may exercise supplemental jurisdiction over additional claims that arise out of a common nucleus of operative fact, the addition of a party cannot defeat complete diversity. Here, the defendant is a resident of State B. Because the passenger is also a resident of State B, adding the passenger's claim would defeat complete diversity.

At the close of a civil negligence trial in federal court, the judge properly explained to the jury that under the law of respondeat superior, the employer cannot be liable if the employee is not also liable. The jury returned a verdict against the employer but not the employee, even though the plaintiff had presented evidence that the employer's liability solely arose under the respondeat superior doctrine. The plaintiff requested that the jury be polled, and each juror confirmed the verdict as his or her own. Before the jury was dismissed, both the employer and the plaintiff requested that the court resubmit the case to the jury. Can the court grant this request?

A Yes, because the plaintiff had requested that the jury be polled. BYes, because the jury did not follow the court's instructions in reaching its verdict. CNo, because the jury has already rendered its verdict. DNo, because a juror may not impeach her own verdict. Answer choice B is correct. When a jury's verdict does not properly follow the court's instructions, the court may set aside the verdict and order the jury to resume deliberations. In this case, because the court properly instructed the jury as to the law, the court can order the jury to resume deliberations, and would even be within its discretion to order a new trial.

A plaintiff filed a complaint in federal district court alleging that a city police officer had violated the plaintiff's constitutional rights during a traffic stop. The complaint, which asserted claims against the city and a named police officer, was filed the day before the statute of limitations for the claims was set to expire. The named defendants filed an answer, and the parties engaged in expedited discovery. During discovery, the plaintiff learned that he had misidentified the officer in his complaint, and that it was actually the named officer's partner who had stopped him. Discovery also revealed that the named officer's partner was aware of the suit and the fact that he was the intended defendant. Ninety days after filing the complaint, the plaintiff filed a motion to amend his complaint to remove the named police officer and add the officer's partner as a defendant. The officer's partner objected, arguing that the amendment was not timely and the claims were barred by the statute of limitations. Should the court allow the plaintiff to amend his complaint?

A Yes, because the plaintiff is entitled to amend his complaint once as a matter of right. BYes, because the amendment relates back to the date of the original pleading. CNo, because the claims against the new officer are barred by the statute of limitations. DNo, because the amendment was not made within 14 days of service of the answer. Answer choice B is correct. A court should freely grant leave to amend a pleading when justice so requires. Under Rule 15, an amendment to a pleading that changes the named defendant relates back to the date of the original pleading if it asserts essentially the same claim or defense, and the new party knew or should have known of the potential action and received notice within 120 days after the filing of the original complaint.

A plaintiff filed a complaint in federal district court against a federal law enforcement officer for violation of the plaintiff's Fourth Amendment rights in arresting the plaintiff. The complaint sought damages from the officer personally. In accordance with the law of the forum state, the plaintiff served the complaint and summons on the officer by certified mail, return receipt requested. Was this service of process sufficient?

A Yes, because the plaintiff sent the summons as well as the complaint. BYes, because the plaintiff is seeking damages from the officer personally. CNo, because the court's subject matter jurisdiction is not based on diversity. DNo, because the United States has not been served with process. Answer choice D is correct. Even though a plaintiff is pursuing an action against a U.S. officer or employer in his individual capacity, the plaintiff is required to also serve process on the United States if the basis for the action is conduct that occurred in connection with duties performed on behalf of the United States.

A homeowner filed a complaint against a contractor for damages arising from negligent construction of an addition to the homeowner's house. The complaint was timely and properly filed in federal district court sitting in diversity jurisdiction. Nineteen days after the contractor filed his answer, the homeowner learned that material supplied by a manufacturer and used by the contractor was defective. The next day, without leave of the court or the contractor's agreement, the homeowner amended the complaint to add the manufacturer as a defendant in this action. The manufacturer filed a motion to dismiss the complaint against her. The manufacturer pointed out that she had no knowledge of the action nor reason to know of it prior to the homeowner's amendment of the complaint, and that the controlling statute of limitations had expired after the original complaint was filed and before the amended complaint was filed. Should the court grant the manufacturer's motion to dismiss?

A Yes, because the statute of limitations had expired before the amended complaint was filed. BYes, because the amended complaint was filed without leave of the court or the contractor's agreement. CNo, because the relation back doctrine applies to this amendment. DNo, because the amendment was filed within the required time period for amending a complaint as of right.

In a civil action properly removed from state court to federal district court based on diversity jurisdiction, a jury of eight persons was empanelled. During the trial, the court excused a juror for good cause. The case was submitted to the jury and the remaining jurors returned a verdict for the plaintiff by a vote of 6-to-1. The law of the state in which the court is located requires a verdict to be rendered by a jury of at least six persons in a civil lawsuit and permits a verdict to be reached by a vote of 5-to-1. Can the defendant successfully challenge this verdict?

A Yes, because the verdict was returned by a jury with seven members. BYes, because the verdict was not unanimous. CNo, because at least six members of the jury voted in favor of the verdict. DNo, because this is a diversity action and state law permits a verdict by a lesser majority. Answer choice B is correct. Under the Federal Rules of Civil Procedure, a jury verdict must be unanimous, unless the parties stipulate otherwise. Here, the verdict, while rendered by a substantial majority, does not satisfy the unanimity requirement.

Upon the death of the insured, the named beneficiary of the $150,000 life insurance policy on the insured sought payment of the insurance proceeds. The personal representative of the insured's estate, contending that the beneficiary had exerted undue influence over the insured, demanded payment of the insurance proceeds to the insured's estate. The insurer filed an interpleader action pursuant to Rule 22 of the Federal Rules of Civil Procedure, naming both the beneficiary and the personal representative as defendants. In its complaint, the insurer alleged that, since the insured committed suicide, the insurer was not obligated to pay out the insurance proceeds to either defendant. The beneficiary and the personal representative are citizens of the same state; the insurer is a citizen of another state. Does the court have subject matter jurisdiction over this action?

A Yes, because there is no amount-in-controversy requirement in a Rule 22 interpleader action. BYes, because diversity jurisdiction exists. CNo, because both of the claimants are citizens of the same state. DNo, because the insurer has alleged that it is not liable to either defendant. Answer choice B is correct. A federal action based on interpleader pursuant to Rule 22 requires that the general rules for subject matter jurisdiction be satisfied. For diversity jurisdiction, the citizenship of the party bringing the action must be completely diverse from that of the claimants and the statutory amount in controversy must be met. The claimants need not be diverse among themselves. Here, the amount in controversy exceeds $75,000 and the two defendants (claimants) are from a different state than the plaintiff-insurer. Consequently, both the amount-in-controversy and the diversity-of-citizenship requirements are met and the court has subject matter jurisdiction based on diversity.

A manufacturer properly brought a product liability action in federal district court based on a design defect in a machine used by the manufacturer in producing a product. The suit named both the maker of the machine and the inventor of the machine as defendants. All three parties were citizens of different states. After the manufacturer won a judgment against both defendants, the manufacturer properly sought and obtained satisfaction of the judgment from the machine maker. The machine maker has now sued the inventor in federal district court for indemnification of $50,000 of the amount paid to the manufacturer. The inventor has asserted that the machine maker's failure to raise the indemnification claim in the prior action precludes this lawsuit. Should the court rule in favor of the inventor?

A Yes, because this claim arose out of the same transaction or occurrence as the manufacturer's claim. BYes, because the machine maker failed to raise the indemnification claim in the prior action. CNo, because the court would not have had subject-matter jurisdiction over the indemnification claim. DNo, because a cross-claim is never mandatory. Answer choice D is correct. A party is never required to assert a cross-claim against a co-party. Consequently, the machine maker was not required in the prior action to plead that the inventor was liable to the maker if the maker was found liable to the manufacturer. Therefore, the maker may now raise this indemnification claim in the current action.

A corporation that sold pollution control chemicals and related equipment brought suit in federal district court under diversity jurisdiction against a former employee for unfair competition under state law. The corporation's suit was based on information it had been given that the former employee was going to open her own pollution control business in the state using confidential information of the corporation. After filing the complaint, the corporation moved for preliminary injunctive relief to restrain and enjoin the former employee from competing with the corporation. The former employee responded to the motion, disputing the facts as alleged by the corporation. After a full hearing on the motion for a preliminary injunction, the court concluded that it was unclear whether the corporation would ultimately prevail in its suit and that discovery would likely be needed for the corporation to be able to make its claim. Should the court issue a preliminary injunction based on the corporation's motion?

A Yes, if the corporation is likely to suffer irreparable harm in the absence of a preliminary injunction. BYes, because the former employee had notice and a hearing. CNo, because it is unclear whether the corporation will succeed on the merits of its claim. DNo, because a court with diversity jurisdiction cannot hear a claim for preliminary injunctive relief. Answer choice C is correct. A plaintiff seeking a preliminary injunction must establish that: (i) she is likely to succeed on the merits; (ii) she is likely to suffer irreparable harm in the absence of relief; (iii) the balance of equities is in her favor; and (iv) the injunction is in the best interest of the public.

As permitted by State A law, a decedent's only heir personally filed a wrongful death action against a defendant. The decedent was domiciled in State A at the time of his death, which occurred in State A. The decedent's heir (the plaintiff) is domiciled in State B. The action was filed in the State A federal court for the district in which the defendant is domiciled. The complaint seeks damages of $250,000, excluding interest and court costs, and alleges that, "Jurisdiction is founded upon diversity, the plaintiff being a resident of State B, and the defendant a resident of State A." On these facts, is there any basis on which the defendant could properly challenge the plaintiff's complaint?

A Yes, the complaint fails to properly allege the subject matter jurisdiction of the court. BYes, the personal representative of a decedent's estate is deemed a citizen of the same state as the decedent. CNo, the plaintiff and the defendant are domiciled in different states and the jurisdictional threshold has been met. DNo, the complaint satisfies the Federal Rules of Civil Procedure requirements of notice pleading. Answer choice A is correct. In order for a federal trial court to have jurisdiction based on diversity of citizenship, no plaintiff can be a citizen of the same state as any defendant. The complaint must contain a statement of the grounds for the court's jurisdiction. Because citizenship, not residency, is the basis for diversity jurisdiction, the complaint here fails to allege the court's subject matter jurisdiction and could properly be challenged by the defendant.

A plaintiff filed suit in federal district court under diversity jurisdiction, asserting a state-law claim against two co-defendants for civil fraud in a business transaction. Each defendant's attorney indicated that she sought to depose eight different witnesses, for a total of 16 witness depositions by the defense. The plaintiff declined to stipulate that the defendants can take more than 10 depositions. Do the Federal Rules of Civil Procedure allow all 16 depositions?

A Yes, the defense may automatically depose all 16 witnesses. BYes, the defense may depose the 16 witnesses with the approval of the court. CNo, the defense may not depose the 16 witnesses without the plaintiff's agreement. DNo, the defense is subject to an absolute limit of 10 depositions in an action based on diversity jurisdiction. Answer choice B is correct. Without leave of court, the plaintiffs, the defendants, and the third-party defendants, each as a group, are limited to 10 depositions by oral or written examination. Unless the parties agree to the deposition, leave of court must be obtained to exceed the 10-deposition limit. If leave of court is granted, a group may exceed the 10-deposition limit.

A plaintiff properly filed suit for negligence in a federal district court against a defendant corporation. The plaintiff had worked at the corporation's industrial warehouse for the past 24 years and was recently diagnosed with a fatal disease associated with a chemical that had been used at the corporation's warehouse. Use of the chemical violated a federal law enacted to protect the health of workers. The plaintiff is aware that a supplier to the defendant corporation has documents in its possession indicating that the defendant corporation used the chemical because it was less expensive than other alternative chemicals and had purposefully concealed its use.

A Yes, the plaintiff may subpoena the documents from the supplier. BYes, the plaintiff may serve a request for production on the supplier. CNo, the plaintiff must join the supplier in the suit in order to request the documents. DNo, the plaintiff cannot obtain documents from the supplier, a non-party. Answer choice A is correct. Nonparties may be compelled to produce documents pursuant to a subpoena. Answer choice B is incorrect. A request for production of documents can only be made on a party.

A plaintiff sued a defendant corporation for negligence based on product liability in federal district court sitting in diversity jurisdiction. The plaintiff had been injured when her lawnmower, manufactured by the defendant, shot debris into her eye, permanently blinding her. The defendant corporation asserted the defense of assumption of the risk. At the time of the federal district court case, the forum state's highest court had never addressed whether assumption of the risk could apply as a defense in a product liability action. The federal district court, relying on lower court rulings in the state, predicted that the forum state's highest court would not allow the defense of assumption of the risk and refused to instruct the jury on the defense. The defendant lost the case and appealed to the appropriate U.S. court of appeals. During the pendency of the appeal, the forum state's highest court issued an opinion holding that the defense of assumption of the risk applies in product liability cases in the forum state. Should the U.S court of appeals allow the defendant to raise the issue of assumption of the risk on appeal?

A es, because it is bound by the new decision of the forum state's highest court. BYes, if it independently agrees with the decision of the forum state's highest court. CNo, because at the time the federal district court decided the matter, the forum state's highest court had never addressed the matter. DNo, because the federal district court had properly relied on lower court cases from the forum state to reach its decision on the matter. Answer choice A is correct. When a forum state's highest court has not spoken on an issue, a federal district court sitting in diversity must try to determine how the state's highest court would rule on the issue, if it did consider it. To make this determination, the federal court will generally look to any lower state-court decisions that have considered the issue and will follow a lower court's view, unless it believes that the highest state court would not follow it. If, after a federal district court action has been completed, the state's highest court rules on an issue in a way that is different from the way the district court predicted, then a federal appeals court is bound by the state court's ruling. In this case, the forum state's highest court had not yet considered the application of assumption of the risk to product liability actions when the federal district court made its ruling. Thus, the district court was permitted to follow the rulings of the state's lower courts on the issue to predict that the state's highest court would not allow the defense. However, since the forum state's highest court subsequently issued an opinion holding that the defense of assumption of the risk applied to product liability actions, the U.S. court of appeals is bound by that ruling. Thus, it should allow the defendant to raise the issue on appeal.

A plaintiff brought an action against a defendant in federal district court sitting in diversity jurisdiction for civil damages resulting from an alleged battery by the defendant. By agreement of the parties, the case was tried by the court without a jury. The plaintiff presented a witness who testified that the defendant had hit the plaintiff with no provocation. The defendant presented two other witnesses, who each testified that the plaintiff had threatened the defendant prior to the defendant striking the plaintiff. The trial judge found in favor of the plaintiff, specifically finding that the testimony of the plaintiff's witness was more persuasive to her than the testimony of the two defense witnesses. On appeal, a three-judge panel of the court of appeals reviewed the trial transcript. The appellate judges all, in good faith, agreed that the testimony of the two defense witnesses was more persuasive than the testimony of the plaintiff's witness and that it is likely that the trial judge erred in her findings. There are no other issues of fact or law in the case. Should the court of appeals overturn the trial court's decision?

A es, because they reviewed the transcript de novo for factual errors and believed that the trial judge was incorrect in her ruling. BYes, because they would be overturning a finding of law, not fact. CNo, because the appellate court cannot consider factual findings by the trial court judge on appeal. DNo, because they would be overturning a finding of fact, not law. Answer choice D is correct. The district court made a finding of fact, not law, in deciding to believe the testimony of the plaintiff's witness over the defendant's witnesses. Findings of fact cannot be set aside unless clearly erroneous, and due regard must be given to the opportunity of the trial court to judge the credibility of the witnesses. Here, the trial judge observed the demeanor of the witnesses, and it was within the trial judge's province to decide whom to believe. Even if the appellate judges think that the trial judge may have been wrong, that would not be enough to overturn the trial court's decision that is not clearly erroneous. Answer choice A is incorrect. Only findings of law, not findings of fact, are reviewed de novo by the court of appeals.

The plaintiff, a State X citizen, is a stock broker in a State X branch of a brokerage firm that has its principal place of business in State Y. The plaintiff, whose performance consistently places her in the top quarter of her firm, earns $75,000 a year. She did not get promoted to a partnership at her firm, which would guarantee her a minimum compensation of $200,000 annually. Instead, she was offered a "Senior Broker" position at $100,000 a year. The plaintiff believes that the firm's decision was motivated by her ethnic background. The local office has 100 partners (96 are State X citizens), but only one partner is of the plaintiff's same ethnic background, even though over 10 percent of the local population is. She sued the firm in State X's federal district court and alleged that the firm's actions violated both (1) state contract law and (2) a federal statute granting employees a cause of action in federal court against employers who allegedly discriminate based on race or ethnicity. The firm objected on the ground that the court lacks jurisdiction. What is the most likely result?

The court has federal-question jurisdiction to decide the federal law claim, but not the state contract claim, which must be brought in state court because of federalism concerns. BThe court has diversity jurisdiction to decide the state contract claim, and federal-question jurisdiction to decide the federal law claim. CThe court has federal-question jurisdiction to decide the federal law claim, and can assert supplemental jurisdiction over the state contract claim. DThe court must decline to exercise jurisdiction until the state courts resolve the plaintiff's state law contract claim. Answer choice C is correct. The court has federal-question jurisdiction because the case arises under a federal employment discrimination statute that grants the plaintiff a cause of action. The court does not have diversity jurisdiction over the state-law claim, however. Although the amount in controversy exceeds the $75,000 statutory threshold because the plaintiff, by being denied a partnership in the firm, has suffered an annual loss of $100,000 in compensation, the citizenship test is not met. Because the firm is a partnership, it is a citizen of each state that a partner is a citizen for purposes of determining diversity jurisdiction. Because 96 partners of the local office of the firm are citizens of the same state as the plaintiff, diversity of citizenship does not exist. Nevertheless, the court can exercise supplemental jurisdiction over the state contract claim because it arises out of "a common nucleus of operative fact" (the firm's decision not to promote the plaintiff to a partnership) as the federal claim.


Set pelajaran terkait

Dichotomous Key practice questions

View Set